You are on page 1of 63

Best Approach

Circle
(Sheet)

By Mathematics Wizard
Manoj Chauhan Sir (IIT Delhi)
No. 1 Faculty on Unacademy,
Exp. More than 14 Years in
Top Most Coachings of Kota
Produced AIR 1, 12, 18 & 24
Maths IIT-JEE ‘Best Approach’ (MC SIR) Circle
KEY CONCEPTS (CIRCLE)
STANDARD RESULTS :
1. EQUATION OF A CIRCLE IN VARIOUS FORM:
(a) The circle with centre (h, k) & radius ‘r’ has the equation;
(x  h)2 + (y  k)2 = r2.
(b) The general equation of a circle is x2 + y2 + 2gx + 2fy + c = 0 with centre as:
(g, f) & radius = g2  f 2  c .
Remember that every second degree equation in x & y in which coefficient of
x2 = coefficient of y2 & there is no xy term always represents a circle.
If g2 + f 2  c > 0  real circle.
g2 + f 2  c = 0  point circle.
2 2
g + f  c < 0 imaginary circle.
Note that the general equation of a circle contains three arbitrary constants, g, f & c which corresponds
to the fact that a unique circle passes through three non collinear points.
(c) The equation of circle with (x1 , y1) & (x2 , y2) as its diameter is :
(x  x1) (x  x2) + (y  y1) (y  y2) = 0.
Note that this will be the circle of least radius passing through (x1 , y1) & (x2 , y2).
2. INTERCEPTS MADE BY A CIRCLE ON THE AXES :
The intercepts made by the circle x2 + y2 + 2gx + 2fy + c = 0 on the co-ordinate axes are
2 g2  c & 2 f 2  c respectively..
NOTE :
If g2  c > 0  circle cuts the x axis at two distinct points.
If g2 = c  circle touches the x-axis.
If 2
g <c  circle lies completely above or below the x-axis.
3. POSITION OF A POINT w.r.t. A CIRCLE :
The point (x1 , y1) is inside, on or outside the circle x2 + y2 + 2gx + 2fy + c = 0.
according as x12 + y12 + 2gx1 + 2fy1 + c  0 .
Note : The greatest & the least distance of a point A from a circle
with centre C & radius r is AC + r & AC  r respectively.
4. LINE & A CIRCLE :
Let L = 0 be a line & S = 0 be a circle. If r is the radius of the circle & p is the length of the
perpendicular from the centre on the line, then :
(i) p > r  the line does not meet the circle i. e. passes out side the circle.
(ii) p = r  the line touches the circle.
(iii) p < r  the line is a secant of the circle.
(iv) p = 0  the line is a diameter of the circle.
5. PARAMETRIC EQUATIONS OF A CIRCLE :
The parametric equations of (x  h)2 + (y  k)2 = r2 are :
x = h + r cos  ; y = k + r sin  ;   <    where (h, k) is the centre,
r is the radius &  is a parameter.
Note that equation of a straight line joining two point  &  on the circle x2 + y2 = a2 is
     
x cos + y sin = a cos .
2 2 2

Get 10% Instant Discount On Unacademy Plus [Use Referral Code: MCSIRLIVE] 2
Maths IIT-JEE ‘Best Approach’ (MC SIR) Circle
6. TANGENT & NORMAL :
(a) The equation of the tangent to the circle x2 + y2 = a2 at its point (x1 , y1) is,
x x1 + y y1 = a2. Hence equation of a tangent at (a cos , a sin ) is ;
x cos  + y sin  = a. The point of intersection of the tangents at the points P() and Q() is
   
a cos 2 , a sin 2 .
   
cos 2
cos 2
(b) The equation of the tangent to the circle x2 + y2 + 2gx + 2fy + c = 0 at its point (x1 , y1) is
xx1 + yy1 + g (x + x1) + f (y + y1) + c = 0.
(c) y = mx + c is always a tangent to the circle x2 + y2 = a2 if c2 = a2 (1 + m2) and the point of contact
 2 2

is   a m , a  .
 c c
(d) If a line is normal / orthogonal to a circle then it must pass through the centre of the circle. Using
this fact normal to the circle x2 + y2 + 2gx + 2fy + c = 0 at (x1 , y1) is
y1  f
y  y1 = (x  x1).
x1  g
7. A FAMILY OF CIRCLES :
(a) The equation of the family of circles passing through the points of intersection of two circles
S1 = 0 & S2 = 0 is : S1 + K S2 = 0 (K  1).
(b) The equation of the family of circles passing through the point of intersection of a circle
S = 0 & a line L = 0 is given by S + KL = 0.
(c) The equation of a family of circles passing through two given points (x1 , y1) & (x2 , y2) can be written
in the form :
x y 1
(x  x1) (x  x2) + (y  y1) (y  y2) + K x1 y 1 1 = 0 where K is a parameter..
x2 y2 1
(d) The equation of a family of circles touching a fixed line y  y1 = m (x  x1) at the fixed point (x1 , y1) is
(x  x1)2 + (y  y1)2 + K [y  y1  m (x  x1)] = 0 , where K is a parameter.
In case the line through (x1 , y1) is parallel to y - axis the equation of the family of circles touching it
at (x1 , y1) becomes (x  x1)2 + (y  y1)2 + K (x  x1) = 0.
Also if line is parallel to x - axis the equation of the family of circles touching it at
(x1 , y1) becomes (x  x1)2 + (y  y1)2 + K (y  y1) = 0.
(e) Equation of circle circumscribing a triangle whose sides are given by L1 = 0 ; L2 = 0 &
L3 = 0 is given by ; L1L2 +  L2L3 +  L3L1 = 0 provided co-efficient of xy = 0 & co-efficient of
x2 = co-efficient of y2.
(f) Equation of circle circumscribing a quadrilateral whose side in order are represented by the lines
L 1 = 0, L 2 = 0, L 3 = 0 & L 4 = 0 is L 1 L 3 +   L 2 L 4 = 0 provided co-efficient of
x2 = co-efficient of y2 and co-efficient of xy = 0.
8. LENGTH OF A TANGENT AND POWER OF A POINT :
The length of a tangent from an external point (x1 , y1) to the circle
S  x2 + y2 + 2gx + 2fy + c = 0 is given by L = x 12  y 12  2 gx 1  2 f1 y  c = S1 .
Square of length of the tangent from the point P is also called THE POWER OF POINT w.r.t. a circle.
Power of a point remains constant w.r.t. a circle.
Note that : power of a point P is positive, negative or zero according as the point ‘P’ is outside, inside
or on the circle respectively.

Get 10% Instant Discount On Unacademy Plus [Use Referral Code: MCSIRLIVE] 3
Maths IIT-JEE ‘Best Approach’ (MC SIR) Circle
9. DIRECTOR CIRCLE:
The locus of the point of intersection of two perpendicular tangents is called the DIRECTOR CIRCLE of the
given circle. The director circle of a circle is the concentric circle having radius equal to 2 times the
original circle.
10. EQUATION OF THE CHORD WITH A GIVEN MIDDLE POINT :
The equation of the chord of the circle S  x2 + y2 + 2gx + 2fy + c = 0 in terms of its mid point
x1  g
M (x1, y1) is y  y1 =  (x  x1). This on simplication can be put in the form
y1  f
xx1 + yy1 + g (x + x1) + f (y + y1) + c = x12 + y12 + 2gx1 + 2fy1 + c
which is designated by T = S1.
Note that : the shortest chord of a circle passing through a point ‘M’ inside the circle,
is one chord whose middle point is M.
11. CHORD OF CONTACT :
If two tangents PT1 & PT2 are drawn from the point P (x1, y1) to the circle
S  x2 + y2 + 2gx + 2fy + c = 0, then the equation of the chord of contact T1T2 is :
xx1 + yy1 + g (x + x1) + f (y + y1) + c = 0.

REMEMBER :
(a) Chord of contact exists only if the point ‘P’ is not inside .
2LR
(b) Length of chord of contact T1 T2 = .
R 2 L2

R L3
(c) Area of the triangle formed by the pair of the tangents & its chord of contact = 2 2
R L
Where R is the radius of the circle & L is the length of the tangent from (x1, y1) on S = 0.
 2R L 
(d) Angle between the pair of tangents from (x1, y1 ) = tan1  
 L2  R 2 
 
where R = radius ; L = length of tangent.
(e) Equation of the circle circumscribing the triangle PT1 T2 is :
(x  x1) (x + g) + (y  y1) (y + f) = 0.
(f) The joint equation of a pair of tangents drawn from the point A (x1 , y1) to the circle
x2 + y2 + 2gx + 2fy + c = 0 is : SS1 = T2.
Where S  x2 + y2 + 2gx + 2fy + c ; S1  x12 + y12 + 2gx1 + 2fy1 + c
T  xx1 + yy1 + g(x + x1) + f(y + y1) + c.
12. POLE & POLAR :
(i) If through a point P in the plane of the circle , there be drawn any straight line to meet the circle
in Q and R, the locus of the point of intersection of the tangents at Q & R is called the POLAR
OF THE POINT P ; also P is called the POLE OF THE POLAR.
(ii) The equation to the polar of a point P (x1 , y1) w.r.t. the circle x2 + y2 = a2 is given by
xx 1 + yy 1 = a 2 , & if the circle is general then the equation of the polar becomes
xx1 + yy1 + g (x + x1) + f (y + y1) + c = 0. Note that if the point (x1 , y1) be on the circle then the
chord of contact, tangent & polar will be represented by the same equation.
 Aa 2 Ba 2 
(iii) Pole of a given line Ax + By + C = 0 w.r.t. any circle x2 + y2 = a2 is   , .

 C C 

Get 10% Instant Discount On Unacademy Plus [Use Referral Code: MCSIRLIVE] 4
Maths IIT-JEE ‘Best Approach’ (MC SIR) Circle
(iv) If the polar of a point P pass through a point Q, then the polar of Q passes through P.
(v) Two lines L1 & L2 are conjugate of each other if Pole of L1 lies on L2 & vice versa Similarly two points
P & Q are said to be conjugate of each other if the polar of P passes through Q & vice-versa.
13. COMMON TANGENTS TO TWO CIRCLES :
(i) Where the two circles neither intersect nor touch each other , there are FOUR common tangents,
two of them are transverse & the others are direct common tangents.
(ii) When they intersect there are two common tangents, both of them being direct.
(iii) When they touch each other :
(a) EXTERNALLY : there are three common tangents, two direct and one is the tangent at the
point of contact .
(b) INTERNALLY : only one common tangent possible at their point of contact.
(iv) Length of an external common tangent & internal common tangent to the two circles is given by:
Lext = d 2  ( r1  r2 ) 2 & Lint = d 2  ( r1  r2 ) 2 .
Where d = distance between the centres of the two circles . r1 & r2 are the radii of the two circles.
(v) The direct common tangents meet at a point which divides the line joining centre of circles
externally in the ratio of their radii.
Transverse common tangents meet at a point which divides the line joining centre of circles
internally in the ratio of their radii.
14. RADICAL AXIS & RADICAL CENTRE :
The radical axis of two circles is the locus of points whose powers w.r.t. the two circles are equal. The
equation of radical axis of the two circles S1 = 0 & S2 = 0 is given ;
S1  S2 = 0 i.e. 2 (g1  g2) x + 2 (f1  f2) y + (c1  c2) = 0.
NOTE THAT :
(a) If two circles intersect, then the radical axis is the common chord of the two circles.
(b) If two circles touch each other then the radical axis is the common tangent of the two circles at
the common point of contact.
(c) Radical axis is always perpendicular to the line joining the centres of the two circles.
(d) Radical axis need not always pass through the mid point of the line joining the centres of the two
circles.
(e) Radical axis bisects a common tangent between the two circles.
(f) The common point of intersection of the radical axes of three circles taken two at a time is
called the radical centre of three circles.
(g) A system of circles , every two which have the same radical axis, is called a coaxial system.
(h) Pairs of circles which do not have radical axis are concentric.
15. ORTHOGONALITY OF TWO CIRCLES :
Two circles S1= 0 & S2= 0 are said to be orthogonal or said to intersect orthogonally if the tangents
at their point of intersection include a right angle. The condition for two circles to be orthogonal
is : 2 g1 g2 + 2 f1 f2 = c1 + c2 .
Note :
(a) Locus of the centre of a variable circle orthogonal to two fixed circles is the radical axis between the
two fixed circles .
(b) If two circles are orthogonal, then the polar of a point 'P' on first circle w.r.t. the second circle passes
through the point Q which is the other end of the diameter through P . Hence locus of a point which
moves such that its polars w.r.t. the circles S1 = 0 , S2 = 0 & S3 = 0 are concurrent in a circle which is
orthogonal to all the three circles.

Get 10% Instant Discount On Unacademy Plus [Use Referral Code: MCSIRLIVE] 5
Maths IIT-JEE ‘Best Approach’ (MC SIR) Circle
SOLVED EXAMPLES

1. Find the equation of the circle whose centre is (1, –2) and radius is 4.
Sol. The equation of the circle is (x – 1)2 + (y – (–2))2 = 42
 (x – 1)2 + (y + 2)2 = 16
 x2 + y2 – 2x + 4y – 11 = 0 Ans.

2. Find the equation of the circle which passes through the point of intersection of the lines 3x – 2y – 1 = 0
and 4x + y – 27 = 0 and whose centre is (2, –3).
Sol. Let P be the point of intersection of the lines AB and LM whose equations are respectively
3x – 2y – 1 = 0 .....(i)
and 4x + y – 27 = 0 .....(ii)
Solving (i) and (ii) , we get x = 5, y = 7, So coordinates of P are (5, 7).
Let C (2, –3) be the centre of the circle. Since the circle passes through P, therefore
2 2
CP = radius   5  2    7  3 = radius

 radius = 109 .
2
Hence the equation of the required circle is (x – 2)2 + (y + 3)2 =  109 
3. Find the centre & radius of the circle whose equation is x2 + y2 – 4x + 6y + 12 = 0.
Sol. Comparing it with the general equation x2 + y2 + 2gx + 2fy + c = 0, We have
2g = – 4  g=–2
2f = 6  f=3
& c = 12
2 2 2 2
 centre is (–g, –f) i.e., (2, –3) and radius = g  f  c   2    3  12  1

4. Find the equation of the circle, the coordinates of the end points of whose diameter are (–1, 2) and
(4, –3).
Sol. We know that the equation of the circle described on the line segment joining (x1, y1) and (x2, y2) as
a diameter is (x – x1) (x – x2) + (y – y1) (y – y2) = 0.
Here x1 = –1, x2 = 4, y1 = 2 and y2 = –3
So, the equation of the required circle is
(x + 1) (x – 4) + (y – 2) (y + 3) = 0  x2 + y2 – 3x + y – 10 = 0

5. Find the equation to the circle touching the y-axis at a distance –3 from the origin and intercepting a
length 8 on the x-axis.
Sol. Let the equation of the circle be x2 + y2 + 2gx + 2fy + c = 0.
Since it touches y-axis at (0, –3) and (0, –3) lies on the circle.
 c = f2 .....(i) 9 – 6f + c = 0 .....(ii)
2 2
From (i) and (ii) , we get 9 – 6f + f = 0  (f – 3) = 0  f = 3.
Putting f = 3 in (i) we obtain c = 9.
It is given that the circle x2 + y2 + 2gx + 2fy + c = 0 intercepts length 8 on x-axis
 2 g 2  c  8  2 g 2  9  8  g2 – 9 = 16 g = ± 5
Hence, the required circle is x2 + y2 ± 10x + 6y + 9 = 0.

Get 10% Instant Discount On Unacademy Plus [Use Referral Code: MCSIRLIVE] 6
Maths IIT-JEE ‘Best Approach’ (MC SIR) Circle
6. Find the parametric equations of the circle x2 + y2 – 4x – 2y + 1 = 0.
Sol. We have : x2 + y2 – 4x – 2y + 1 = 0  (x2 – 4x) + (y2 – 2y) = –1
 2 2
(x – 2) + (y – 1) = 2 2

So, the parametric equations of this circle are


x = 2 + 2 cos  , y = 1 + 2 sin .

7. Find the equations of the following curves in cartesian form. Also, find the centre and radius of the circle
x = a + c cos  , y = b + c sin 
x a yb
Sol. We have : x = a + c cos  y = b + c sin   cos = , sin =
c c
2 2
 x a   yb  2 2
     = cos  + sin   (x – a)2 + (y – b)2 = c2
 c   c 
Clearly, it is a circle with centre at (a, b) and radius c.

8. Discuss the position of the points (1, 2) and (6, 0) with respect to the circle x2 + y2 – 4x + 2y – 11 = 0
Sol. We have x2 + y2 – 4x + 2y – 11 = 0 or S = 0, where S = x2 + y2 – 4x + 2y – 11.
For the point (1, 2), we have S1 = 12 + 22 – 4 × 1 + 2 × 2 – 11 < 0
For the point (6, 0), we have S2 = 62 + 02 – 4 × 6 + 2 × 0 – 11 > 0
Hence, the point (1, 2) lies inside the circle and the point (6, 0) lies outside the circle.

9. For what value of c will the line y = 2x + c be a tangent to the circel x2 + y2 = 5 ?


Sol. We have : y = 2x + c or 2x – y + c = 0..... (i) and x2 + y2 = 5 ..... (ii)
If the line (i) touches the circle (ii), then
Length of the from the centre (0, 0) = radius of circle (ii)

2 0  0  c c
  5   5
2
2
2   1 5

c
  5  c=±5
5
Hence, the line (i) touches the circle (ii) for c = ± 5

10. Find the equation of the tangent to the circle x2 + y2 – 30x + 6y + 109 = 0 at (4, –1).
Sol. Equation of tangent is

 x  4   y   1 
4x + (–y) – 30    6  + 109 = 0
 2   2 
or 4x – y – 15x – 60 + 3y – 3 + 109 = 0 or –11x + 2y + 46 = 0
or 11x – 2y – 46 = 0
Hence, the required equation of the tangent is 11x – 2y – 46 = 0.

11. Find the equation of tangents to the circle x2 + y2 – 6x + 4y – 12 = 0 which are parallel to the line
4x + 3y + 5 = 0
Sol. Given circle is x2 + y2 – 6x + 4y – 12 = 0 .....(i)
and given line is 4x + 3y + 5 = 0 .....(ii)
Centre of circle (i) is (3, –2) and its radius is 5. Equation of any line
4x + 3y + k = 0 parallel to the line (ii) .....(iii)

Get 10% Instant Discount On Unacademy Plus [Use Referral Code: MCSIRLIVE] 7
Maths IIT-JEE ‘Best Approach’ (MC SIR) Circle
If line (iii) is tangent to circle, (i) then
| 4.3  3(2)  k |
= 5 or |6 + k| = 25
4 2  32
or 6 + k = ± 25  k = 19, – 31
Hence equation of required tangents are 4x + 3y + 19 = 0 and 4x + 3y – 31 = 0

12. Find the equation of the normal to the circle x2 + y2 – 5x + 2y – 48 = 0 at the point (5, 6).
Sol. The equation of the tangent to the circle x2 + y2 – 5x + 2y – 48 = 0 at (5, 6) is
 x 5  y6
5x + 6y – 5    2  – 48 = 0  10x + 12y – 5x – 25 + 2y + 12 – 96 = 0
 2   2 
 5x + 14y – 109 = 0
5 14
 Slope of the tangent =   Slope of the normal =
14 5
Hence, the equation of the normal at (5, 6) is
y – 6 = (14/5)(x – 5)  14x – 5y – 40 = 0

13. Find the equation of the pair of tangents drawn to the circle x2 + y2 – 2x + 4y = 0 from the point (0, 1)
Sol. Given circle is S = x2 + y2 – 2x + 4y = 0 .....(i)
Let P  (0, 1)
For Point P, S1 = 02 + 12 – 2.0 + 4.1 = 5
Clearly P lies outside the circle
and T x . 0 + y . 1 – (x + 0) + 2(y + 1)
i.e. T –x + 3y + 2.
Now equation of pair of tangents from P(0, 1) to circle (1) is SS1 = T2
or 5(x2 + y2 – 2x + 4y) = (–x + 3y + 2)2
or 5x2 + 5y2 – 10x + 20y = x2 + 9y2 + 4 – 6xy – 4x + 12y
or 4x2 – 4y2– 6x + 8y + 6xy – 4 = 0
or 2x2 – 2y2 + 3xy – 3x + 4y – 2 = 0 ..... (ii)
Note : Separate equation of pair of tangents : From (ii) 2x + 3(y – 1) x – (2y2 – 4y + 2) = 0
2

2
3  y  1  9  y  1  8  2y 2  4y  2 
 x=
4

or 4x + 3y – 3 = ± 25y 2  50y  25 = ± 5(y – 1)


 Separate equations of tangents are x + 2y – 2 = 0 and 2x – y + 1 = 0.

14. Find the length of the tangent drawn from the point (5, 1) to the circle x2 + y2 + 6x – 4y – 3 = 0.
Sol. Given circle is x2 + y2 + 6x – 4y – 3 = 0 .....(i)
Given point is (5, 1), Let P = (5, 1)
Now length of the tangent from P(5, 1) to circle (i) = 52  12  6.5  4.1  3  7

15. Find the equation of director circle of the circle (x – 2)2 + (y + 1)2 = 2.
Sol. Centre & radius of given circle are (2, –1) & 2 respectively..
Centre and radius of the director circle will be (2, –1) & 2 × 2 = 2 respectively..
 equation of director circle is (x – 2)2 + (y + 1)2 = 4

Get 10% Instant Discount On Unacademy Plus [Use Referral Code: MCSIRLIVE] 8
Maths IIT-JEE ‘Best Approach’ (MC SIR) Circle
 x2 + y2 – 4x + 2y + 1 = 0
16. Find the equation of the chord of contact of the tangents drawn from (1, 2) to the circle
x2 + y2 – 2x + 4y + 7 = 0
Sol. Given circle is x2 + y2 – 2x + 4y + 7 = 0 ..... (i)
Let P = (1, 2)
For point P(1, 2), x2 + y2 – 2x + 4y + 7 = 1 + 4 – 2 + 8 + 7 = 18 > 0
Hence point P lies outside the circle
For point P(1, 2), T = x . 1 + y . 2 – (x + 1) + 2(y + 2) + 7
i.e., T = 4y + 10
Now equation of the chord of contact of point P(1, 2) w.r.t. circle (i) will be
4y + 10 = 0 or 2y + 5 = 0

17. Tangents are drawn to the circle x2 + y2 = 12 at the points where it is met by the circle
x2 + y2 – 5x + 3y – 2 = 0 ; find the point of intersection of these tangents.
Sol. Given circles are S1 x2 + y2 – 12 = 0 .....(i)
and S2 = x2 + y2 – 5x + 3y – 2 = 0 .....(ii)
Now equation of common chord of circle (i) and (ii) is
S1 – S2 = 0 i.e., 5x – 3y – 10 = 0
Let this line meet circle (i) [or (ii) at A and B]
Let the tangents to circle (i) at A and B meet at P(), then AB will be the chord of contact of the
tangents to the circle (i) from P, therefore equation of AB will be

x+ y– 12 = 0
Now lines (iii) and (iv) are same, therefore, equations (iii) and (iv) are identical
  12 18
     6, –
5 3 10 5
 18 
Hence P =  6,  
 5
18. Find the equation of the polar of the point (2, –1) with respect to the circle x2 + y2 – 3x + 4y – 8 = 0.
Sol. Given circle is x2 + y2 – 3x + 4y – 8 = 0 .....(i)
Given point is (2, –1) let P = (2, –1). Now equation of the polar of point P with respect to circle (i)
 x  2   y 1 
x.2 + y(–1) – 3    4 – 8 = 0
 2   2 
or 4x – 2y – 3x – 6 + 4y – 4 – 16 = 0 or x + 2y – 26 = 0

19. Find the pole of the line 3x + 5y + 17 = 0 with respect to the circle x2 + y2 + 4x + 6y + 9 = 0.
Sol. Given circle is x2 + y2 + 4x + 6y + 9 = 0 .....(i)
and given line is 3x + 5y + 17 = 0 .....(ii)
Let P() be the pole of line (ii) with respect to circle (i)
Now equation of polar of point P() with respect to circle (i) is
x+ y+ 2(x +) + 3(y + ) + 9 = 0
or (+ 2)x + (+ 3)y + 2+ 3+ 9 = 0 .....(iii)

Get 10% Instant Discount On Unacademy Plus [Use Referral Code: MCSIRLIVE] 9
Maths IIT-JEE ‘Best Approach’ (MC SIR) Circle
Now lines (ii) and (iii) are same, therefore,
  2   3 2  3  9
 
3 5 17
(i) (ii) (iii)
From (i) and (ii), we get
5+ 10 = 3 + 9 or 5 – 3= –1 .....(iv)
From (i) and (iii), we get
17+ 34 = 6+ 9+ 27 or 11– 9= –7 .....(v)
Solving (iv) & (v), we get = 1, = 2
Hence required pole is (1, 2).

20. Find the equation of the chord of the circle x2 + y2 + 6x + 8y – 11 = 0, whose middle point is (1, –1).
Sol. Equation of given circle is S x2 + y2 + 6x + 8y – 11 = 0
Let L  (1, –1)
For point L(1, –1), S1 = 12 + (–1)2 + 6.1 + 8(–1) – 11 = – 11 and
T x.1 + y(–1) + 3(x + 1) + 4(y – 1) – 11 i.e., T 4x + 3y – 12
Now equation of the chord of circle (i) whose middle point is L (1, –1) is
T = S1 or 4x + 3y – 12 = – 11 or 4x + 3y – 1 = 0
Second Method
4  1 3
Let C be the centre of the given circle, then C (–3, –4), L (1, –1) slope of CL = 
3  1 4
 Equation of chord of circle whose middle point is L, is
4
 y + 1 = – (x – 1) [ chord is perpendicular to CL]
3
or 4x + 3y – 1 = 0

21. Examine if the two circles x2 + y2 – 2x – 4y = 0 and x2 + y2 – 8y – 4 = 0 touch each other externally or
internally.
Sol. Given circles are x2 + y2 – 2x – 4y = 0 .....(i)
2 2
and x + y – 8y – 4 = 0 .....(ii)
Let A and B be the centres and r1 and r2 the radii of circles (i) and (ii) respectively, then
A (1, 2), B  (0, 4), r1 = 5 , r2 = 2 5
2 2
Now AB = 1  0    2  4  5 and r1+ r2 = 3 5 , |r1 – r2| = 5
Thus AB = |r1 – r2|, hence the two circles touch each other internally.

22. Obtain the equation of the circle orthogonal to both the circles x2 + y2 + 3x – 5y + 6 = 0 and
4x2 + 4y2 – 28x + 29 = 0 and whose centre lies on the line 3x + 4y + 1 = 0.
Sol. Given circles are x2 + y2 + 3x – 5y + 6 = 0 .....(i)
and 4x2 + 4y2 – 28x + 29 = 0
29
or x2 + y2 – 7x +  0. .....(ii)
4
Let the required circle be x2 + y2 + 2gx + 2fy + c = 0 .....(iii)
Since circle (iii) cuts circles (i) and (ii) orthogonally
3  5
 2g   + 2f    = c + 6 or 3g – 5f = c + 6 .....(iv)
2  2

Get 10% Instant Discount On Unacademy Plus [Use Referral Code: MCSIRLIVE] 10
Maths IIT-JEE ‘Best Approach’ (MC SIR) Circle

 7 29 29
and 2g    + 2f.0 = c + or –7g = c + .....(v)
 2 4 4
5
From (iv) & (v) , we get 10g – 5f = 
4
or 40g – 20f = – 5 .....(vi)
Given line is 3x + 4y = – 1 .....(vii)
Since centre (–g, – f) of circle (iii) lies on line (vii),
 –3g – 4g = –1 .....(viii)
1
Solving (vi) & (viii), we get g = 0, f =
4
29
 from (5), c =   from (iii), required circle is
4
1 29
x2 + y2 + y– =0 or 4(x2 + y2) + 2y – 29 = 0
2 4

23. Find the co-ordinates of the point from which the lengths of the tangents to the following three circles be
equal.
3x2 + 3y2 + 4x – 6y – 1 = 0
2x2 + 2y2 – 3x – 2y – 4 = 0
2x2 + 2y2 – x + y – 1 = 0
Sol. Here we have to find the radical centre of the three circles. First reduce them to standard form in which
coefficients of x2 and y2 be each unity. Subtracting in pairs the three radical axis are
17 5 3 3 11 5 1
xy 0 ; x  y   0 ;  x y 0
6 3 2 2 6 2 6
 16 31 
solving any two, we get the point   ,   which satisfies the third also. This point is called the
 21 63 
radical centre and by definition the length of the tangents from it to the three circles are equal.

24. Find the equations of the circles passing through the points of intersection of the circles
x2 + y2 – 2x – 4y – 4 = 0 and x2 + y2 – 10x – 12y + 40 = 0 and whose radius is 4.
Sol. Any circle through the intersection of given circles is S1 + S2 = 0
or (x2 + y2 – 2x – 4y – 4) + (x2 + y2 – 10x – 12y + 40) = 0 .....(i)
(1  5 ) (2  6) 40  4
or (x2 + y2) – 2 x–2 y+ =0
1  1  1 
2 2
r = g  f  c = 4, given
2 2
1  5  
 2  6  
40  4
 16 = 2 2
1    1    1 
16(1 + 2+ ) = 1 + 10+ 25+ 4 + 24 + 36– 40– 40+ 4 + 4
or 16 + 32 + 16 = 21– 2+ 9 or 5– 34– 7 = 0
 (– 7) (5+ 1) = 0  = 7, – 1/5
Putting the values of in (i) the required circles are
2x2 + 2y2 – 18x – 22y + 69 = 0 and x2 + y2 – 2y – 15 = 0

Get 10% Instant Discount On Unacademy Plus [Use Referral Code: MCSIRLIVE] 11
Maths IIT-JEE ‘Best Approach’ (MC SIR) Circle
25. Find the equations of circles which touche 2x – y + 3 = 0 and pass through the points of intersection of
the line x + 2y – 1 = 0 and the circle x2 + y2 – 2x + 1= 0.
Sol. The required circle by S +L = 0 is
x2 + y2 – 2x + 1 + (x + 2y – 1) = 0
or x2 + y2 – x(2 –) + 2y + (1 –) = 0
centre (–g, –f) is [(2 – )/2, – ]
r= g2  f 2  c
2 1
= 2   / 4   2  1    
5 2    / 2  5
2
Since the circle touches the line 2x – y + 3 = 0 therefore perpendicular from centre is equal to radius
2.[(2   ) / 2]  ( )  3  
 5 or 5 = ± .5  = ± 2
 5 2 2
Putting the values of in (i) the required circles are
x2 + y2 + 4y – 1 = 0
x2 + y2 – 4x – 4y + 3 = 0.

26. Find the equation of circle passing through the points A(1, 1) & B(2, 2) and whose radius is 1.
Sol. Equation of AB is x – y = 0
 equation of circle is
(x – 1) (x – 2) + (y – 1) (y – 2) + (x – y) = 0
or x2 + y2 + (– 3) x – (+ 3) y + 4 = 0
2 2
   3 
   3 4
radius =
4 4
2 2

But radius = 1 (given) ;     3 


   3 4 1
4 4
or (– 3)2 + (+ 3)2 – 16 = 4.
or 2= 2
or = ± 1
 Equation of circle is
x2 + y2 – 2x – 4y + 4 = 0
& x2 + y2 – 4x – 2y + 4 = 0 Ans.

27. Find the equation of the circle passing through the point (2, 1) and touching the line x + 2y – 1 = 0 at the
point (3, – 1).
Sol. Equation of circle is
(x – 3)2 + (y + 1)2 + (x + 2y – 1) = 0
Since it passes through the point (2, 1)
1 + 4 + (2 + 2 – 1) = 0
 = –5/3
 circle is
(x – 3)2 + (y + 1)2 – 5/3 (x + 2y – 1) = 0
 3x2 + 3y2 – 23x – 4y + 35 = 0 Ans.

Get 10% Instant Discount On Unacademy Plus [Use Referral Code: MCSIRLIVE] 12
Maths IIT-JEE ‘Best Approach’ (MC SIR) Circle
28. Find the equation of circle circumscribing the triangle whose sides are 3x – y – 9 = 0, 5x – 3y – 23 = 0
& x + y – 3 = 0.
A
X

Sol. L1 : 3x – y –9 = 0 L2 : 5x –3y –23 = 0

B C
L3 : x +y –3 = 0

L1L2 + L2L3 + L1L3 = 0


(3x – y – 9) (5x – 3y – 23) + (5x – 3y – 23) (x + y – 3) + (3x – y – 9)(x + y – 3) = 0
(15x2 + 3y2 – 14xy – 114x + 50y + 207) + (5x2 – 3y2 + 2xy – 38x – 14y + 69)
+ (3x2 – y2 + 2xy – 18x – 6y + 27) = 0
(5+ 3+ 15) x2 + (3 – 3–)y2 + xy (2+ 2– 14) – x (114 + 38+ 18)
+ y(50 – 14– 6) + (207 + 69+ 27) = 0 .....(i)
coefficient of x2 = coefficient of y2
 5+ 3+ 15 = 3 – 3– 
++ 12 = 0
2+ + 3 = 0 .....(ii)
coefficient of xy = 0
 2+ 2– 14 = 0
 – 7 = 0 .....(iii)
Solving (ii) and (iii) , we have
= – 10, = 17
Putting these values of & in equation (i) , we get
2x2 + 2y2 – 5x + 11y – 3 = 0

29. Find the locus of the points of intersection of the tangents to the circle x = r cos  , y = r sin at points
whose parametric angles differ by /3.
Sol. All such points P satisfying the given condition will be equidistant from the origin O (see fig.)
Hence the locus of P will be a circle centred at the origin, having radius equal to
r 2r B
OP =  P
 3
cos  
6
O A
4 2
Therefore, equation of the required locus is x2 + y2 = r .
3

30. If – 3l2 – 6l – 1 + 6m2 = 0, find the equation of the circle for which lx + my + 1 = 0 is a tangent.
Sol. The given expression can be written as
3l  1
6(l 2 + m2) = 9l 2 + 6l + 1 i.e.,  6
l 2  m2
From this expression we can infer that the perpendicular distance of the point (3, 0) from the line
lx + my + 1 = 0 is 6 .
This proves that the given line is a tangent to the circle (x – 3)2 + y2 = 6.

Get 10% Instant Discount On Unacademy Plus [Use Referral Code: MCSIRLIVE] 13
Maths IIT-JEE ‘Best Approach’ (MC SIR) Circle
31. Prove that x2 + y2 = a2 and (x – 2a)2 + y2 = a2 are two equal circles touching each other. Find the
equation of circle (or circles ) of the same radius touching both the circles.
Sol. Given circles are
x2 + y2 = a2 .....(1)
and 2 2
(x – 2a) + y = a 2 .....(2)
Let A and B be the centres and r1 and r2 the radii of the circles (1) and (2) respectively. Then
A  (0, 0), B  (2a, 0), r1 = a, r2 = a
2
Now AB =  0  2a   02 = 2a = r1 + r2
Hence the two circles touch each other externally.
Let the equation of the circle having same radius 'a' and touching the circles (1) and (2) be
(x – )2 + (y –)2 = a2 .....(3)
Its centre C is () and radius r3 = a
Since circle (3) touches the circle (1)
AC = r1 + r3 = 2a. [Here AC |r1 – r3| as r1 – r3 = a – a = 0]
 AC = 4a   +  = 4a
2 2 2  

Again since circle (3) touches the circle (2)


BC = r2 + r3
 (2a –)2 + = (a + a)2
 – 4a 
 4a2 – 4a = 0
 = a we get = ± 3 a.
Hence, the required circles are (x – a)2 + (y  a 3 )2 = a2
or x2 + y2 – 2ax 2 3 ay + 3a2 = 0

32. If the curves ax2 + 2hxy + by2 + 2gx + 2fy + c = 0 and Ax2 + 2Hxy + By2 + 2Gx + 2Fy + C = 0 intersect
 a  b    A  B
at four concyclic points, prove that .
h H
Sol. Equation of a curve passing through the intersection points of the given curves can be written as
(ax2 + 2hxy + by2 + 2gx + 2fy + c) + (Ax2 + 2Hxy + By2 + 2Gx + 2Fy + C) = 0 .....(1)
2
If this curve must be a circle, then coeff. of x = coeff. of y2

ba
i.e., (a + A) = (b + B) gives = .....(2)
AB
and coeff. of xy = 0
h
i.e., 2(h + H) = 0 given = – .....(3)
H
Equating the two values of , we get the desired result.

33. Let S  x2 + y2 + 2gx + 2fy + c = 0 be a given circle. Find the locus of the foot of the perpendicular
drawn from the origin upon any chord of S which subtends right angle at the origin.

A
P(h,k)
B

Get 10% Instant Discount On Unacademy Plus [Use Referral Code: MCSIRLIVE] 14
Maths IIT-JEE ‘Best Approach’ (MC SIR) Circle

AB is a variable chord such that = AOB = .
2
Let P(h, k) be the foot of the perpendicular drawn from origin upon AB. Equation of the chord AB is
h
y–k = (x – h)
k
i.e., hx + ky = h2 + k2 .....(1)
Equation of the pair of straight lines passing through the origin and the intersection point of the given
circle
x2 + y2 + 2gx + 2fy + c = 0 .....(2)
and the variable chord AB is
2
 hx  ky   hx  ky 
x2 y2
+ + 2(gx + fy)  2 2 
 c 2 2 
0 .....(3)
 h k   h k 
If equation (3) must represent a pair of perpendicular lines, then we have coeff. of x2 + coeff. of y2 = 0
   
2gh ch 2 2fk ck 2
i.e.,  1  2   
 1  0
 h  k 2  h 2  k 2  2   h 2  k 2  h 2  k 2 2 
   
Putting (x, y) in place of (h, k) gives the equation of the required locus as
c
x2 + y2 + gx + fy + = 0.
2
34. The line Ax + By + C = 0 cuts the circle x2 + y2 + gx + fy + c = 0 at P and Q.
The line A'x + B'y + C' = 0 cuts the circle x2 + y2 + g'x + f'y + c' = 0 at R and S.
g  g ' f  f ' c  c'
If P, Q, S are concyclic, show that A B C 0
A' B' C'
Sol. Equation of a circle through P and Q is x2 + y2 + gx + fy + c + (Ax + By + C) = 0
i.e., x2 + y2 + (g + A) x + (f + B)y + (c + C) = 0 .....(1)
2 2
and equation of a circle through R and S is x + y + g'x + f 'y + c' +  (A'x + B'y + C') = 0
x2 + y2 + (g' + A') y + (f ' + B') + (c' + C') = 0 .....(2)
If P, Q, R and S are concyclic points, then equations (1) and (2) must represent the same circle.
g  A f  B c  C
Equating the ratio of the coefficients, we have 1 =  
g ' A ' f ' B' c' C'
i.e A – ' + g – g' = 0 .....(3)
B – B' + f – f' = 0 .....(4)
and C – C' + c – c' = 0 .....(5)
A A ' g  g '
Eliminating and from equation (3), (4) and (5), we have B B' f  f ' = 0
C C ' c  c'

g  g ' f  f ' c  c'


or A B C  0 [Interchanging rows by columns and then interchanging the second and
A' B' C'
the third row]

Get 10% Instant Discount On Unacademy Plus [Use Referral Code: MCSIRLIVE] 15
Maths IIT-JEE ‘Best Approach’ (MC SIR) Circle
Aliter : Let the given circles be S1  x2 + y2 + gx + fy + c = 0 .....(1)
2 2
and S2 x +y + g'x + f 'y + c' = 0 .....(2)
If S be the required circle, then according to the given condition
Ax + By + C = 0 is the radical axis of S1, S
and A'x + B'y + C' = 0 is the radical axis of S2, S
while (g – g')x + (f – f ')y + (c – c') = 0 is the radical axis of S1,S2.
Since the radical axes of three circles taken in pairs are concurrent, therefore, we have
g  g ' f  f ' c  c'
A B C  0 which is the desired result.
A' B' C'
35. Circles are drawn passing through the origin O to intersect the coordinate axes at point P and Q such that
m  OP + n. OQ is a constant. Show that the circles pass through a fixed point.
Sol. Equation of a circle passing through the origin and having X and Y intercepts equal to a and b respectively
is x2 + y2 – ax – by = 0 .....(1)
According to the given condition, we have
ma + nb = k (constant)
k  ma
i.e., b  .....(2)
n
 k  ma 
Putting the above value of b in equation (1) , we have x2 + y2 – ax –   y= 0
 n 
i.e., {n(x2 + y2) – ky} – a (nx – my) = 0
Which represents the equation of a family of circles passing through the intersection points of the circle
n (x2 + y2) – ky = 0 .....(3)
and the line
nx – my = 0 .....(4)
 mk nk 
Solving equation (3) and (4), gives the coordinates of the fixed point as  2 2
, 2 2 
.
m n m n 

p q
36. P(p, q) is a point on a circle passing through the origin and centred at C  ,  . If two distinct chords
2 2
can be drawn from P such that these chords are bisected by the X-axis, then show that p2 > 8q2.
p q
Sol. It can be seen that the given points P(p, q), C  ,  and the origin are collinear which implies that line
2 2
OP where O is the origin is a diameter of the given circle. Therefore, equation of the given circle is
x(x – p) + y(y – q) = 0 Y
i.e x2 + y2 – px – qy = 0 .....(1)
Let M(a, 0) be the mid-point of a chord AP (see fig.) Then, we have P(p,q)
CM  AP / 2)C
,q
(p/2
q
q O 0)
X
i.e., slope of CM × slope of AP = –1 2   1 M
(a ,
p
a pa A
2
i.e., q2 + (p – 2a) (p – a) = 0 i.e., 2a2 – 3pa + p2 + q2 = 0 .....(2)

Get 10% Instant Discount On Unacademy Plus [Use Referral Code: MCSIRLIVE] 16
Maths IIT-JEE ‘Best Approach’ (MC SIR) Circle
Equation (2) which is a quadratic equation in a shows that there will be two real and distinct values of a
if the discriminant is > 0
i.e., if (3p)2 – 4 × 2(p2 + q2) > 0
i.e., if p2 > 8q2
which is the desired result.

Aliter. Equation of the given circle is


x2 + y2 – px – qy = 0 .....(1)
Equation of any line through P(p, q) can be written as
y – q = m (x – p) (where m is a variable)
y   mp  q 
i.e., x .....(2)
m
putting the value of x from equation (2) in equation (1) will give the ordinate of the intersection points of
2
 y   mp  q   2  y   mp  q  
the line and the given circle as    y  p   qy  0
 m   m 
i.e., {y + (mp – q)}2 + m2y2 – mp{y + (mp – q)} – m2qy = 0
i.e., (1 + m2)y2 + {2(mp – q) – mp – m2q}y + (mp – q)2 – mp(mp – q) = 0
i.e., (1 + m2)y2 + (pm – 2q – qm2)y – q(mp – q) = 0 .....(3)
The above equation gives the Y coordinates of the intersection points of the chord and the given circle.
According to the given condition, the mid-point of this intercept lies on the X-axis, therefore we have
sum of the roots of equation (3) = 0
i.e., pm – 2q – qm2 = 0
i.e., qm2 – pm + 2q =0 .....(4)
The above equation shows that there will be two real and distinct values of m if p2 > 8q2 which is the
desired result.

37. Prove that the square of the tangent that can be drawn from any point on one circle to another circle is
equal to twice the product of perpendicular distance of the point from the radical axis of two circles and
distances between their centres.
Sol. Let us choose the circles, as S1 x2 + y2 – a2 = 0 .....(1)
2 2
and S2  (x – b) + y – c = 02 .....(2)
Let P (a cos, a sin) be any point on circle S1. The length of the tangent from P to circle S2, is given by
PT2 = S2(a cos , a sin ) = (acos – b)2 + (a sin )2 – c2 = a2 + b2 – c2 – 2ab cos
The distance between the centres of S1 and S2 is
C1C2 = b
The radical axis of S1 and S2, is 2bx – a2 – b2 + c2 = 0
[equation (1) – equation (2)]
The perpendicular distance of P from the radical axis, is
| 2b  a cos    a 2  b 2  c 2 |
PM =
2b
Now, we have
| 2ab cos   a 2  b 2  c 2 |
2.PM. C1C2 = 2b. = |a2 + b2 – c2 – 2ab cos| = PT2 which proves the
2b
desired result.

Get 10% Instant Discount On Unacademy Plus [Use Referral Code: MCSIRLIVE] 17
Maths IIT-JEE ‘Best Approach’ (MC SIR) Circle

38. Consider a family of circles passing through the intersection point of the lines 3 (y – 1) = x – 1 and
y – 1 = 3 (x – 1) and having its centre on the acute angle bisector of the given lines. Show that the
common chords of each member of the family and the circle x2 + y2 + 4x – 6y + 5 = 0 are concurrent.
Find the point of concurrency.
Sol. The given lines 3 (y – 1) = x – 1 .....(1)
y – 1 = 3 (x – 1) .....(2)
intersect at the point (1, 1)
Rewriting the equation of the given lines such that their constant terms are both positive, we have
x– 3y+ 3 –1=0 .....(3)
and – 3x+y+ 3 –1=0 .....(4)
Here, we have
(product of coeff's of x) + (product of coeff's of y) = – 3 – 3 = – ve quantity which implies that the
acute angle between the given lines contains the origin.
Therefore, equation of the acute angle bisector of the given lines is
x  3y  3  1  3x  y  3  1

2 2
i.e., y=x
Any point on the above bisector can be chose as () and equation of any circle passing through
(1, 1) and having centre at (, ) is
(x –)2 + (y –)2 = (1 –)2 + (1 – )2 .....(6)
i.e., 2 2
x + y – 2x – 2y + 4– 2 = 0
The common chord of the given circle
x2 + y2 + 4x – 6y + 5 = 0 .....(7)
and the circle represented by equation (6) is
(4 + 2) x + (2– 6)y + (7 – 4) = 0
i.e., (4x – 6y + 7) + 2(x + y – 2) = 0 .....(8)
Which represents a family of straight lines passing through the intersection point of the lines
4x – 6y + 7 = 0 .....(9)
and x+y–2=0 .....(10)
1 3
Solving equation (9), (10) gives the coordinates of the fixed point as  ,  .
2 2

39. Find the range of value of  for which the variable line 3x + 4y – = 0 lies between the circles
x2 + y2 – 2x – 2y + 1 = 0 and x2 + y2 – 18x – 2y + 78 = 0 without intercepting a chord on either circle.
Sol. The given circle
S1 x2 + y2 – 2x – 2y + 1 = 0 .....(1)
has centre C1 (1, 1) and radius r1 = 1 Y
The other given circle
S2 x2 + y2 – 18x – 2y + 78 = 0 .....(2) M1
has centre C2 (9, 1) and radius r2 = 2.
According to the required condition, we have
C1 C2
C1M1 r1 X
O
|3 4|
i.e., 1 M2
32  4 2

Get 10% Instant Discount On Unacademy Plus [Use Referral Code: MCSIRLIVE] 18
Maths IIT-JEE ‘Best Approach’ (MC SIR) Circle
i.e., ( – 7) 5 [C1 lies below the line  (7 – ) is a –ve quantity]
i.e., 12 i.e.,
| 27  4  |
2
32  42
i.e., (31– )  10 [ C2 lies below the line]
 (31 –) is a + ve quantity]
i.e., 21
Hence, the permissible values of are 12  21.

40. Point P having integral coordinates lies on x2 + y2 = 1 and x2 + y2 + 2x + 4y + 1 = 0. A chord through


P meets the two circles at A and B. Find the equation of the chord PAB if PA and PB subtend equal
angles at the centres of the respective circles.
Sol. Equation of the given circles are S1  x2 + y2 – 1 = 0 .....(1)
and S2 x2 + y2 + 2x + 4y + 1 = 0 .....(2)
Subtracting equation (2) from equation (1) we have
x = – (2y + 1) .....(3)
2
Putting in equation (1), we have (2y + 1) + y = 12

i.e., 5y2 + 4y = 0 gives y = 0, – 4/5


and the corresponding values of x = – 1, 3/5.

Thus the intersection point of circles S1 and S2, having integral coordinates, is P (–1, 0).
From the fig., we can see that if PA and PB subtend equal angles at C1 and C2 respectively, then
PA : PB = C1A : C2B = 1 : 2
Equation of a line through P can be chosen as
y = m (x + 1) .....(4)
Solving equations (1) and (4) for the intersection point () (say) , we have
x2 + m2(x + 1)2 = 1
i.e., (1 + m2) x2 + 2m2x + (m2 – 1) = 0
whose one root is x = –1 since one of the intersection point is P(–1, 0)

m2 1 1  m2
Thus, we have –1 ×  gives 
1 m2 1 m2
Solving equation (2) and (4) for the intersection point B(, ) (say), we have
x2 + m2(x + 1)2 + 2x + 4m(x + 1) + 1 = 0
i.e., (1 + m2)x2 + (2m2 + 4m + 2)x + (m2 + 4m + 1) = 0
whose one root is x = –1 since one of the intersection point is P(–1, 0).

m 2  4m  1  m 2  4m  1 
Thus, we have –1 ×  = given  =  2 
1  m2  1 m 
Now, using the condition PA : PB = 1 : 2, we have 2 +  = –3
i.e., 2(1 – m2) – (m2 + 4m + 1) = –3(1 + m2)
gives m = 1
Hence, equation of the required chord is y = 1(x + 1).

Get 10% Instant Discount On Unacademy Plus [Use Referral Code: MCSIRLIVE] 19
Maths IIT-JEE ‘Best Approach’ (MC SIR) Circle
41. Curves ax2 + 2hxy + by2 – 2gx – 2fy + c = 0 and a'x2 – 2hxy + (a' + a – b)y2 – 2g'x – 2f 'y + c = 0
 g ' g f '  f 
intersect at four concyclic point A, B, C and D. If P is the point  , A2 + PB2
 prove that PA
 a ' a a '  a 
2
+ PC = 3PD . 2

Sol. Equation of a curve passing through the intersection points of the given curves
ax2 + 2hxy + by2 – 2gx – 2fy + c = 0 .....(1)
and 2 2
a'x – 2hxy + (a' + a – b)y – 2g'x – 2f 'y + c = 0 .....(2)
can be written as {a'x2 – 2hxy + (a' + a – b)y2 – 2g'x – 2f'y + c}
+ {ax2 + 2hxy + by2 – 2gx – 2fy + c} = 0
i.e., (a' + a)x2 + 2h( – 1)xy + (a' + a – b + b)y2
–2(g' + g)x – 2(f ' + f)y + (1 + )c = 0 .....(3)
According to the given condition equation (3) must represent a circle, therefore,
we have coeff. of x2 = coeff. of y2
i.e., a' + a = a' + a – b + b
i.e., (a – b) = a – b
gives  = 1 and coeff. of xy = 0
i.e., –1=0
gives  = 1
The identical values prove that the curve is a circle.
Putting the above value of  in equation (3) gives the equation of the circle passing through the intersection
points of the curves represented by equation (1) and (2) as
(a' + a)(x2 + y2) – 2 (g' + g)x – 2(f' + f)y + 2c = 0
 g ' g f '  f 
which has its centre at the point  , .
 a ' a a '  a 
We can see that the conditions of the given point P is the same as the centre of the circle passing through
the points A, B, C and D. Therefore, we have PA2 = PB2 = PC2 = PD2 = radius of the circle which gives
the desired result PA2 + PB2 + PC2 = 3PD2.

42. A is one of the points of intersection of two given circles. A variable line through A meets the two circles
again at point P and Q. Show that the locus of the mid-point of P and Q is also a circle passing through
A.
Sol. Let us choose the intersection point A as the origin and the radical axis of the circles, as the Y-axis
(see fig.). Then the equation of the circles can be chosen as
S1  x2 + y2 – 2g1x – 2fy = 0 .....(1)
and S2  x2 + y2 – 2g2x – 2fy = 0 .....(2)
Equation of a variable line through A can be written as y = mx.
Putting in equation (1), we have
x2(1 + m2) – 2(g1 + mf)x = 0
2(g1  mf )
gives x = 0,
1  m2
Putting in equation (2), we have x2(1 + m2) – 2(g2 + mf)x = 0
2(g 2  mf )
gives x = 0,
1  m2
2(g1  mf ) 2(g 2  mf )
Thus, we have P  (x1, mx1) where x1 = 2 and Q  (x2, mx2) where x2 =
1 m 1  m2
If M(h, k) be the mid-point of PQ, then 2h = x1 + x2

Get 10% Instant Discount On Unacademy Plus [Use Referral Code: MCSIRLIVE] 20
Maths IIT-JEE ‘Best Approach’ (MC SIR) Circle
i.e., h(1 + m2) = g1 + g2 + 2mf .....(3)
and 2k = m(x1 + x2)
i.e., k(1 + m2) = m(g1 + g2 + 2mf) .....(4)
k
Dividing equation (4) by equation (3), we have m =
h
Putting the above value of m in equation (3), we have
 k2  2kf
h 1  2  = g1 + g2 + i.e., h2 + k2 = (g1 + g2)h + 2fk
 h  h
Putting (x, y) in place of (h, k) gives the equation of the required locus, as
x2 + y2 – (g1 + g2)x – 2fy = 0
which is a circle passing through A(0, 0).

43. Q is a fixed point and S is a fixed circle. A variable chord through Q meets the circle S at point A and B.
Find the locus of a point P on this chord such that QA, QP, QB are in
(a) arithmetic progression
(b) geometric progression
(c) harmonic progression
Sol. Let us choose the line joining Q and the centre of the circle S as the X-axis and the centre of the circle as
the origin (see fig).

Let the coordinates of the fixed point Q be (, 0) and the equation of the fixed circle S be
x2 + y2 = a2 .....(1)
Let  be the inclination of a variable line through Q. The coordinates of any point on this line can be
chosen as ( + r cos , r sin ). If this point also lies on the circle S, then putting the above coordinates
in equation (1),
we have
( + r cos )2 + (r sin )2 = a2
i.e., r2 + (2 cos )r + 2 – a2 = 0 .....(2)
The roots of the above equation, say r1, r2 are the distance QA and QB. Thus, we have
QA + QB = r1 + r2 = – 2 cos 
QA  QB = r1r2 = 2 – a2
Let P(h, k) be the point whose locus is to be found. If the distance QP is denoted by r, then we have
h =  + r cos , k = r sin 
r1  r2
(a) If QP is the A.M. of QA and QB, then r = –  cos [from equation (2)]
2
Thus, we have h =  –  cos2  =  sin2  .....(3)
k = –  cos  sin  .....(4)
k2
Now, we have =  cos2  .....(5)
h

Get 10% Instant Discount On Unacademy Plus [Use Referral Code: MCSIRLIVE] 21
Maths IIT-JEE ‘Best Approach’ (MC SIR) Circle
Adding equations (3) and (4), we have h2 + k2 = h
Hence, the required locus is x2 + y2 – x = 0 which is a circle.
(b) If QP is the G.M. of QA and QB. then

r= r1r2  2  a 2 [from equation (2)]

Thus, we have h =  –  2  a 2 cos , k =  2  a 2 sin 


Eliminating , we have (h – )2 + k2 = 2 – a2
i.e., h2 + k2 – 2h + 2 = 0
Hence, the required locus is x2 + y2 – 2x = 0 which is a circle.
2r1r2 2  a 2
(c) If QP is the H.M. of QA and QB, then r = 
r1  r2  cos 

2  a 2  2  a2 
Thus, we have h =  – , k =   cos   sin 
  
From the first equation above , is eliminated. h = a2
Hence, the required locus is x = a2.

44. Tangents are drawn to the circle x2 + y2 = 50 from a point 'P' lying on the x-axis. These tangents meet the
y-axis at points 'P1' and 'P2'. Possible coordinates of 'P' so that area of triangle PP1P2 is minimum, is/are
(a) (10, 0) (B) ( 10 2 , 0) (C) (–10, 0) (D) (– 10 2 , 0)
Sol. OP = 5 2 sec ,
OP1 = 5 2 cosec ,
100
area (PP1P2) = , area (PP1P2)min = 100
sin 2
  = /4  OP = 10
 P = (10, 0), (–10, 0)
Hence, (A), (C) are correct.

45. Two circles with radii 'r1' and 'r2', r1 > r2  2, touch each other externally. If '' be the angle between the
direct common tangents, then
r r  r r 
(A)  = sin–1  1 2  (B)  = 2sin–1  1 2 
 r1  r2   r1  r2 

r r 
(C)  = sin–1  1 2  (D) none of these
 r1  r2 
r1  r2
Sol. sin  =
r1  r2

r r 
  = 2sin–1  1 2 
 r1  r2 
Hence (B) is correct

Get 10% Instant Discount On Unacademy Plus [Use Referral Code: MCSIRLIVE] 22
Maths IIT-JEE ‘Best Approach’ (MC SIR) Circle
46. If the curves ax2 + 4xy + 2y2 + x + y + 5 = 0 and ax2 + 6xy + 5y2 + 2x + 3y + 8 = 0 intersect at four
concyclic points then the value of a is
(A) 4 (B) –4 (C) 6 (D) –6
Sol. Any second degree curve passing through the intersection of the given curves is
ax2 + 4xy + 2y2 + x + y + 5 +(ax2 + 6xy + 5y2 + 2x + 3y + 8) = 0
If it is a circle, then coefficient of x2 = coefficient of y2 and coefficient of xy = 0
a(1 + ) = 2 + 5 and 4 + 6 = 0
10
2
2  5 2 3
 a= and =   a= 2 = – 4.
1  3 1
3
Hence (B) is correct answer.

47. The chords of contact of the pair of tangents drawn from each point on the line 2x + y = 4 to the circle
x2 + y2 = 1 pass through a fixed point -
 1 1 1 1
(A) (2, 4) (B)   ,   (C)  ,  (D) (–2, –4)
 2 4 2 4
Sol. The chord of contact of tangents from (, ) is
x + y = 1 .....(1)
1 1
Hence, (1) passes through  , 
2 4
Hence (C) is correct answer.

48. Equation of chord AB of circle x2 + y2 = 2 passing through P(2, 2) such that PB/PA = 3, is given by -
(A) x = 3y (B) x = y
(C) y – 2 = 3 (x – 2) (D) none of these
y2 x2
Sol. Any line passing through (2, 2) will be of the form  =r
sin  cos 
When this line cuts the circle x2 + y2 = 2, (r cos  + 2)2 + (2 + r sin )2 = 2
 r2 + 4(sin  + cos ) r + 6 = 0
PB r2
  , now if r1 = , r2 = 3,
PA r1
then 4 = –4(sin  + cos ), 32 = 6  sin 2 = 1 = /4
So required chord will be y – 2 = 1(x – 2)  y = x.
Alternative solution
PA. PB = PT2 = 22 – 2 = 6 .....(1)
PB
=3 .....(2)
PA
From (1) and (2), we have PA = 2 , PB = 3 2
 AB = 2 2 . Now diameter of the circle is 2 2 (as radius is 2)
Hence line passing through the centre  y = x.
Hence (B) is the correct answer.

Get 10% Instant Discount On Unacademy Plus [Use Referral Code: MCSIRLIVE] 23
Maths IIT-JEE ‘Best Approach’ (MC SIR) Circle
49. Equation of a circle S(x, y) = 0, (S, (2, 3) = 16) which touches the line 3x + 4y – 7 = 0 at (1, 1) is given
by
(A) x2 + y2 + x + 2y – 5 = 0 (B) x2 + y2 + 2x + 2y – 6 = 0
(C) x2 + y2 + 4x – 6y = 0 (D) none of these
Sol. Any circle which touches 3x + 4y – 7 = 0 at (1, 1) will be of the form
S(x, y)  (x – 1)2 + (y – 1)2 + (3x + 4y – 7) = 0
Since, S(2, 3) = 16  = 1, so required circle will be
x2 + y2 + x + 2y – 5 = 0
Hence, (A) is the correct answer.

50. If (a, 0) is a point on a diameter of the circle x2 + y2 = 4, then x2 – 4x – a2 = 0 has


(A) exactly one real root in (–1, 0] (B) exactly one real root in [2, 5]
(C) distinct roots greater than –1 (D) distinct roots less than 5
Sol. Since (a, 0) is a point on the diameter of the circle x2 + y2 = 4.
So maximum value of a2 is 4.

Let f(x) = x2 – 4x – a2
Clearly f(–1) = 5 – a2 0 f(2) = – (a2 + 4) < 0
f(0) = – a2 < 0 and f(5) = 5 – a2 > 0
So graph of f(x) will be as shown
Hence (A), (B), (C), (D) are the correct answer.

51. If a circle S(x, y) = 0 touches at the point (2, 3) of the line x + y = 5 and S(1, 2) = 0, then radius of such
circle
1 1
(A) 2 units (B) 4 units (C) units (D) units
2 2
Sol. Desired equation of the circle is
(x – 2)2 + (y – 3)2 + (x + y – 5) = 0
1 + 1 + (1 + 2 – 5) = 0  = 1
x2 – 4x + 4 + y2 – 6y + 9 + x + y – 5 = 0  x2 + y2 – 3x – 5y + 8 = 0
2 2
 3  5 25 9 2 1
x   y  = – 8 +   
 2  2 4 4 4 2
Hence (D) is the correct answer.

52. If P(2, 8) is an interior point of a circle x2 + y2 – 2x + 4y – p = 0 which neither touches nor intersects the
axes, then set for p is -
(A) p < –1 (B) P < –4 (C) p > 96 (D) 
Sol. For internal point p(2, 8), 4 + 64 – 4 + 32 – p < 0  p > 96 and x intercept = 2 1  p therefore
1+p<0
 p < –1 and y intercept 2 4  p  p < – 4
Hence (D) is the correct answer.

53. If two circles (x –1 )2 + (y – 3)2 = r2 and x2 + y2 – 8x + 2y + 8 = 0 intersect in two distinct point then
(A) 2 < r < 8 (B) r < 2 (C) r = 2 (D) r > 2
Sol. Let d be the distance between the centres of two circles of radii r1 and r2.
These circle intersect at two distinct points if |r1 – r2| < d < r1 + r2

Get 10% Instant Discount On Unacademy Plus [Use Referral Code: MCSIRLIVE] 24
Maths IIT-JEE ‘Best Approach’ (MC SIR) Circle
Here, the radii of the two circles are r and 3 and distance between the centres is 5.
Thus, |r – 3| < 5 < r + 3  –2 < r < 8 and r > 2  2 < r < 8
Hence (A) is the correct answer.

54. The common chord of x2 + y2 – 4x – 4y = 0 and x2 + y2 = 16 subtends at the origin an angle equal to
(A) /6 (B) /4 (C) /3 (D) /2
Sol. The equation of the common chord of the circles x + y – 4x – 4y = 0 and x2 + y2 = 16 is x + y = 4
2 2

which meets the circle x2 + y2 = 16 at points A(4, 0) and B(0, 4). Obviously OA  OB.
Hence the common chord AB makes a right angle at the centre of the circle x2 + y2 = 16.
Hence (D) is the correct answer.

55. The number of common tangents that can be drawn to the circle x2 + y2 – 4x – 6y – 3 = 0 and
x2 + y2 + 2x + 2y + 1 = 0 is
(A) 1 (B) 2 (C) 3 (D) 4
Sol. The two circles are
x2 + y2 – 4x – 6y – 3 = 0 and x2 + y2 + 2x + 2y + 1 = 0
Centre : C1  (2, 3), C2  (–1, –1) radii : r1 = 4, r2 = 1
We have C1C2 = 5 = r1 + r2, therefore there are 3 common tangents to the given circles.
Hence (C) is the correct answer.

56. The tangents drawn from the origin to the circle x2 + y2 – 2rx – 2hy + h2 = 0 are perpendicular if
(A) h = r (B) h = –r (C) r2 + h2 = 1 (D) r2 = h2
Sol. The combined equation of the tangents drawn from (0, 0) to
x2 + y2 – 2rx – 2hy + h2 = 0 is
(x2 + y2 – 2rx – 2hy + h2)h2 = (–rx – hy + h2)2
This equation represents a pair of perpendicular straight lines if coeff. of x2 + coeff. of y2 = 0
i.e., 2h2 – r2 – h2 = 0
 r2 = h2 or r = ± h. Hence (A), (B) and (D) are correct answers.

57. The equation(s) of the tangent at the point (0, 0) to the circle, making intercepts of length 2a and 2b units
on the coordinate axes, is(are) -
(A) ax + by = 0 (B) ax – by = 0 (C) x = y (D) none of these
Sol. Equation of circle passing through origin and cutting off intercepts 2a and 2b units on the coordinate axes
is x2 + y2 ± 2ax ± 2by = 0
Hence, (A), (B) are correct answers.

Get 10% Instant Discount On Unacademy Plus [Use Referral Code: MCSIRLIVE] 25
Maths IIT-JEE ‘Best Approach’ (MC SIR) Circle

ELEMENTARY EXERCISES
( SL. LONEY - EX-17 )

Find the equation to the circle

Q.1 Whose radius is 3 and whose centre is (–1, 2).

Q.2 Whose radius is 10 and whose centre is (–5, –6).

Q.3 Whose radius is a + b and whose centre is (a, – b).

Q.4 Whose radius is a 2  b 2 and whose centre is (– a, – b).

Find the coordinates of the centres and the radii of the circles whose equations are

Q.5 x2 + y2 – 4x – 8y = 41

Q.6 3x2 + 3y2 – 5x – 6y + 4 = 0

Q.7 x2 + y2 = k(x + k)

Q.8 x2 + y2 = 2gx – 2fy

Q.9 1  m 2 (x 2  y 2 )  2cx  2mcy  0

Draw the circles whose equations are

Q.10 x2 + y2 = 2ay

Q.11 3x2 + 3y2 = 4x

Q.12 5x2 + 5y2 = 2x + 3y

Q.13 Find the equation to the circle which passes through the points (1, –2) and (4, –3) and which has its
centre on the straight line 3x + 4y = 7.

Q.14 Find the equation to the circle passing through the points (0, a) and (b, h), and having its centre on the
axis of x.

Find the equations to the circles which pass through the points

Q.15 (0, 0), (a, 0) and (0, b)

Q.16 (1, 2), (3, –4) and (5, –6)

Q.17 (1, 1), (2, –1) and (3, 2)


Q.18 (5, 7), (8, 1) and (1, 3)

Q.19 (a, b), (a, – b) and (a + b, a – b)

Get 10% Instant Discount On Unacademy Plus [Use Referral Code: MCSIRLIVE] 26
Maths IIT-JEE ‘Best Approach’ (MC SIR) Circle

Q.20 ABCD is a square whose side is a; taking AB and AD as axes, prove that the equation to the circle
circumscribing the square is x2 + y2 = a (x + y)
Q.21 Find the equation to the circle which passes through the origin and cuts off intercepts equal to 3 and 4
from the axes.
Q.22 Find the equation to the circle passing through the origin and the points (a, b) and (b, a). Find the lengths
of the chords that it cuts off from the axes.
Q.23 Find the equation to the circle which goes through the origin and cuts off intercepts equal to h and k from
the positive parts of the axes.
Q.24 Find the equation to the circle, of radius a, which passes through the two points on the axis of x which are
at a distance b from the origin.
Find the equation to the circle which
Q.25 touches both axis at a distance 5 from the origin.
Q.26 touches both axis and is of radius a.
Q.27 touches both axes and passes through the point ( –2, –3).
Q.28 touches the axis of x and passes through the two points (1, –2) and (3, –4).
Q.29 touches the axis of y at the origin and passes through the point (b, c).
Q.30 touches the axis of x at a distance 3 from the origin and intercepts a distance 6 on the axis of y.
Q.31 Points (1, 0) and (2, 0) are taken on the axis of x, the axes being rectangular. On the line joining these
points an equilateral triangle is described, its vertex being in the positive quadrant. Find the equations to
the circles described on its sides as diameters.
Q.32 If y = mx be the equation of a chord of a circle whose radius is a, the origin of coordinates being one
extremity of the chord and the axis of x being a diameter of the circle, prove that the equation of a circle
of which this chord is the diameter is (1 + m2) (x2 + y2) – 2a (x + my) = 0.
Q.33 Find the equation to the circle passing through the points (12, 43), (18, 39), and (42, 3) and prove that
it also passes through the points (–54, –69) and (–81, –38).
Q.34 Find the equation to the circle circumscribing the quadrilateral formed by the straight lines
2x + 3y = 2, 3x – 2y = 4, x + 2y = 3 and 2x – y = 3.
Q.35 Prove that the equation to the circle of which the points (x1 , y1) and (x2, y2) are the ends of a chord of
a segment containing an angle  is
(x – x1) (x – x2) + (y – y1) (y – y2) ± cot [(x – x1) (y – y2) – (x – x2) (y – y1)] = 0.
Q.36 Find the equations to the circles in which the line joining the points (a, b) and (b, – a) is a chord subtending
an angle of 45° at any point on its circumference.

Get 10% Instant Discount On Unacademy Plus [Use Referral Code: MCSIRLIVE] 27
Maths IIT-JEE ‘Best Approach’ (MC SIR) Circle

( SL. LONEY - EX-18)

Write down the equation of the tangent to the circle

Q.1 x2 + y2 – 3x + 10y = 15 at the point (4, –11).

 11 
Q.2 4x2 + 4y2 – 16x + 24y = 117 at the point  4,   .
 2

Find the equations to the tangents to the circle

Q.3 x2 + y2 = 4 which are parallel to the line x + 2y + 3 = 0

Q.4 x2 + y2 + 2gx + 2fy + c = 0 which are parallel to the line x + 2y – 6 = 0.

Q.5 Prove that the straight line y  x  c 2 touches the circle x2 + y2 = c2, and find its point of contact.

Q.6 Find the condition that the straight line cx – by + b2 = 0 may touch the circle x2 + y2 = ax + by and find
the point of contact.

Q.7 Find whether the straight line x  y  2  2 , touches the circle

x2 + y2 – 2x – 2y + l = 0.

Q.8 Find the condition that the straight line 3x + 4y = k may touch the circle x2 + y2 = 10x.

Q.9 Find the value of p so that the straight line

x cos  + y sin – p = 0

may touch the circle

x2 + y2 – 2ax cos  – 2by sin  – a2 sin2  = 0.

Q.10 Find the condition that the straight line Ax + By + C = 0 may touch the circle

(x – a)2 + (y – b)2 = c2

Q.11 Find the equation to the tangent to the circle x2 + y2 = a2 which


(i) is parallel to the straight line y = mx + c,
(ii) is perpendicular to the straight line y = mx + c,
(iii) passes through the point (b, 0), and
(iv) makes with the axes a triangle whose area is a2.

x y
Q.12 Find the length of the chord joining the points in which the straight line   1 , meet the circle
a b

x2 + y2 = r2

Get 10% Instant Discount On Unacademy Plus [Use Referral Code: MCSIRLIVE] 28
Maths IIT-JEE ‘Best Approach’ (MC SIR) Circle

Q.13 Find the equation to the circles which pass through the origin and cut off equal chords a from the straight
lines y = x and y = –x.

Q.14 Find the equation to the straight lines joining the origin to the points in which the straight line y = rnx + c
cuts the circle

x2 + y2 = 2ax + 2by

Hence find the condition that these points may subtend a right angle at the origin.

Find also the condition that the straight line may touch the circle.

Find the equation to the circle which

Q.15 has its centre at the point (3, 4) and touches the straight line 5x + 12y = 1.

x y
Q.16 touches the axes of coordinates and also the line   1 , the centre being in the positive quadrant.
a b

Q.17 has its centre at the point (1, –3) and touches the straight line 2x – y – 4 = 0.

Q.18 Find the general equation of a circle referred to two perpendicular tangents as axes.

Q.19 Find the equation to a circle of radius r which touches the axis of y at a point distant h from the origin,
the centre of the circle being in the positive quadrant.

Prove also that the equation to the other tangent which passes through the origin is

(r2 – h2) x + 2rhy = 0

Q.20 Find the equation to the circle whose centre is at the point () and which passes through the origin,
and prove that the equation of the tangent at the origin is x + y = 0

Q.21 Two circles are drawn through the points (a, 5a) and (4a, a) to touch the axis of y. Prove that they

40
intersect at an angle tan 1 .
9

Q.22 A circle passes through the points ( –1, 1), (0, 6), and (5, 5). Find the points on this circle the tangents
at which are parallel to the straight line joining the origin to its centre.

Get 10% Instant Discount On Unacademy Plus [Use Referral Code: MCSIRLIVE] 29
Maths IIT-JEE ‘Best Approach’ (MC SIR) Circle

( SL. LONEY - EX-19 )

Find the polar of the point

Q.1 (1, 2) with respect to the circle x2 + y2 = 7.

Q.2 (4, –1) with respect to the circle 2x2 + 2y2 = 11.

Q.3 (–2, 3) with respect to the circle x2 + y2 – 4x – 6y + 5 = 0.

 1 2 2
Q.4  5,   with respect to the circle 3x + 3y – 7x + 8y – 9 = 0.
 2

Q.5 (a, –b) with respect to the circle x2 + y2 + 2ax – 2by + a2 – b2 = 0.

Find the pole of the straight line

Q.6 x + 2y = 1 with respect to the circle x2 + y2 = 5.

Q.7 2x – y = 6 with respect to the circle 5x2 + 5y2 = 9.

Q.8 2x + y + 12 = 0 with respect to the circle x2 + y2 – 4x + 3y – 1 = 0.

Q.9 48x – 54y + 53 = 0 with respect to the circle 3x2 + 3y2 + 5x – 7y + 2 = 0.

Q.10 ax + by + 3a2 + 3b2 = 0 with respect to the circle x2 + y2 + 2ax + 2by = a2 + b2.

Q.11 Tangents are drawn to the circle x2 + y2 = 12 at the points where it is met by the circle

x2 + y2 – 5x + 3y – 2 = 0;

find the point of intersection of these tangents.

Q.12 Find the equation to that chord of the circle x2 + y2 = 81 which is bisected at the point (–2, 3), and its
pole with respect to the circle.

Q.13 Prove that the polars of the point (1, –2) with respect to the circles whose equations are

x2 + y2 + 6y + 5 = 0 and x2 + y2 + 2x + 8y + 5 = 0

coincide; also prove that there is another point the polars of which with respect to these circles are the
same and find its coordinates.

Q.14 Find the condition that the chord of contact of tangents from the point (x', y') to the circle x2 + y2 = a2
should subtend a right angle at the centre.

Q.15 Prove that the distances of two points, P and Q, each from the polar of the other with respect to a circle,
are to one another inversely as the distances of the points from the centre of the circle.

Q.16 Prove that the polar of a given point with respect to any one of the circles x2 + y2 – 2kx + c2 = 0, where
k is variable, always passes through a fixed point, whatever be the value of k.

Get 10% Instant Discount On Unacademy Plus [Use Referral Code: MCSIRLIVE] 30
Maths IIT-JEE ‘Best Approach’ (MC SIR) Circle

Q.17 Tangents are drawn from the point (h, k) to the circle x2 + y2 = a2; prove that the area of the triangle

a(h 2  k 2  a 2 )3/ 2
formed by them and the straight line joining their points of contact is .
h2  k2

Find the lengths of the tangents drawn

Q.18 to the circle 2x2 + 2y2 = 3 from the point (–2, 3).

Q.19 to the circle 3x2 + 3y2 – 7x – 6y = 12 from the point (6, –7).

Q.20 to the circle x2 + y2 + 2bx – 3b2 = 0 from the point (a + b, a – b).

Q.21 Given the three circles

x2 + y2 – 16x + 60 = 0,

3x2 + 3y2 – 36x + 81 = 0,

and x2 + y2 – 16x – 12y + 84 = 0,

find (1) the point from which the tangents to them are equal in length, and (2) this length.

Q.22 The distances from the origin of the centres of three circles x2 + y2 – 2x = c2 (where c is a constant and
 a variable) are in geometrical progression; prove that the lengths of the tangents drawn to them from
any point on the circle x2 + y2 = c2 are also in geometrical progression.

Q.23 Find the equation to the pair of tangents drawn

(1) from the point (11, 3) to the circle x2 + y2 = 65,

(2) from the point (4, 5) to the circle 2x2 + 2y2 – 8x + 12y + 21 = 0

Get 10% Instant Discount On Unacademy Plus [Use Referral Code: MCSIRLIVE] 31
Maths IIT-JEE ‘Best Approach’ (MC SIR) Circle
EXERCISE–I
Q.1 Determine the nature of the quadrilateral formed by four lines 3x + 4y – 5 = 0; 4x – 3y – 5 = 0; 3x + 4y + 5 = 0
and 4x – 3y + 5 = 0. Find the equation of the circle inscribed and circumscribing this quadrilateral.
Q.2 A circle S = 0 is drawn with its centre at (–1, 1) so as to touch the circle x2 + y2 – 4x + 6y – 3 = 0
externally. Find the intercept made by the circle S = 0 on the coordinate axes.
Q.3 The line lx + my + n = 0 intersects the curve ax2 + 2hxy + by2 = 1 at the point P and Q. The circle on PQ
as diameter passes through the origin. Prove that n2(a + b) = l2 + m2.
Q.4 One of the diameters of the circle circumscribing the rectangle ABCD is 4y = x + 7. If A & B are
the points (–3, 4) & (5,4) respectively, then find the area of the rectangle.
Q.5 Let L1 be a straight line through the origin and L2 be the straight line x + y = 1 . If the intercepts made
by the circle x2 + y2  x + 3y = 0 on L1 & L2 are equal, then find the equation(s) which represent L1.
Q.6 A circle passes through the points (–1, 1), (0, 6) and (5, 5). Find the points on the circle the tangents at
which are parallel to the straight line joining origin to the centre.
Q.7 Find the equations of straight lines which pass through the intersection of the lines x  2y  5 = 0,
7x + y = 50 & divide the circumference of the circle x2 + y2 = 100 into two arcs whose lengths are
in the ratio 2 : 1.
Q.8 In the given figure, the circle x2 + y2 = 25 intersects the x-axis at
the point A and B. The line x = 11 intersects the x-axis at the
point C. Point P moves along the line x = 11 above the x-axis
and AP intersects the circle at Q. Find
(i) The coordinates of the point P if the triangle AQB has the maximum area.
(ii) The coordinates of the point P if Q is the middle point of AP.
(iii) The coordinates of P if the area of the triangle AQB is (1/4)th of the area of the triangle APC.
Q.9 A circle is drawn with its centre on the line x + y = 2 to touch the line 4x – 3y + 4 = 0 and pass through
the point (0, 1). Find its equation.
Q.10 A point moving around circle (x + 4)2 + (y + 2)2 = 25 with centre C broke away from it either at the point
A or point B on the circle and moved along a tangent to the circle passing through the point D (3, – 3).
Find the following.
(i) Equation of the tangents at A and B.
(ii) Coordinates of the points A and B.
(iii) Angle ADB and the maximum and minimum distances of the point D from the circle.
(iv) Area of quadrilateral ADBC and the DAB.
(v) Equation of the circle circumscribing the DAB and also the intercepts made by this circle on the
coordinate axes.
Q.11 Find the locus of the mid point of the chord of a circle x2 + y2 = 4 such that the segment intercepted by
the chord on the curve x2 – 2x – 2y = 0 subtends a right angle at the origin.
Q.12 Find the equation of a line with gradient 1 such that the two circles x 2 + y2 = 4 and
x2 + y2 – 10x – 14y + 65 = 0 intercept equal length on it.
Q.13 Find the locus of the middle points of portions of the tangents to the circle x2 + y2 = a2 terminated by the
coordinate axes.
Q.14 Tangents are drawn to the concentric circles x2 + y2 = a2 and x2 + y2 = b2 at right angle to one another.
Show that the locus of their point of intersection is a 3rd concentric circle. Find its radius.
Q.15 Find the equation to the circle which is such that the length of the tangents to it from the points (1, 0),
(2, 0) and (3, 2) are 1,  7 , 2 respectively..

Get 10% Instant Discount On Unacademy Plus [Use Referral Code: MCSIRLIVE] 32
Maths IIT-JEE ‘Best Approach’ (MC SIR) Circle
Q.16 Consider a circle S with centre at the origin and radius 4. Four circles A, B, C and D each with radius
unity and centres (–3, 0), (–1, 0), (1, 0) and (3, 0) respectively are drawn. A chord PQ of the circle S
touches the circle B and passes through the centre of the circle C. If the length of this chord can be
expressed as x , find x.
Q.17 If the variable line 3x – 4y + k = 0 lies between the circles x2 +  y2 – 2x – 2y + 1 = 0 and
x2 + y2 – 16x – 2y + 61 = 0 without intersecting or touching either circle, then the range of k is (a, b)
where a, b  I. Find the value of (b – a).
Q.18 Obtain the equations of the straight lines passing through the point A(2, 0) & making 45° angle with the
tangent at A to the circle (x + 2)2 + (y  3)2 = 25. Find the equations of the circles each of radius 3
whose centres are on these straight lines at a distance of 5 2 from A.
Q.19 A variable circle passes through the point A (a, b) & touches the x-axis; show that the locus of the other
end of the diameter through A is (x  a)2 = 4by.
Q.20 Find the locus of the mid point of all chords of the circle x2 + y2  2x  2y = 0 such that the pair of lines
joining (0, 0) & the point of intersection of the chords with the circles make equal angle with axis of x.
Q.21 A circle with center in the first quadrant is tangent to y = x + 10, y = x – 6, and the y-axis. Let (h, k) be
the center of the circle. If the value of (h + k) = a + b a where a is a surd, find the value of a + b.
Q.22 A circle C is tangent to the x and y axis in the first quadrant at the points P and Q respectively. BC and
AD are parallel tangents to the circle with slope – 1. If the points A and B are on the y-axis while C and
D are on the x-axis and the area of the figure ABCD is 900 2 sq. units then find the radius of the circle.
Q.23 Circles C1 and C2 are externally tangent and they are both internally tangent to the circle C3. The radii of
C1 and C2 are 4 and 10, respectively and the centres of the three circles are collinear. A chord of C3 is
m n
also a common internal tangent of C1 and C2. Given that the length of the chord is where m, n
p
and p are positive integers, m and p are relatively prime and n is not divisible by the square of any
prime, find the value of (m + n + p).
Q.24 Find the equation of the circle passing through the three points (4, 7), (5, 6) and (1, 8). Also find the
coordinates of the point of intersection of the tangents to the circle at the points where it is cut by the
straight line 5x + y + 17 = 0.
Q.25 The line 2x – 3y + 1 = 0 is tangent to a circle S = 0 at (1, 1). If the radius of the circle is 13 . Find the
equation of the circle S.
Q.26 Find the equation of the circle which passes through the point (1, 1) & which touches the circle
x2 + y2 + 4x  6y  3 = 0 at the point (2, 3) on it.
Q.27 Find the equation of the circle whose radius is 3 and which touches the circle x2 + y2 – 4x – 6y – 12=0
internally at the point (–1, – 1).
Q.28 Given that a right angled trapezium has an inscribed circle. Prove that the length of the right angled leg is
the Harmonic mean of the lengths of bases.
Q.29 Let K denotes the square of the diameter of the circle whose diameter is the common chord of the
two circles x2 + y2 + 2x + 3y + 1 = 0 and x2 + y2 + 4x + 3y + 2 = 0
and W denotes the sum of the abscissa and ordinates of a point P where all variable chords of the
curve y2 = 8x subtending right angles at the origin, are concurrent.
and H denotes the square of the length of the tangent from the point (3, 0) on the circle
2x2 + 2y2 + 5y –16 = 0.
Find the value of KWH.
Q.30 Let S1 = 0 and S2 = 0 be two circles intersecting at P (6, 4) and both are tangent to x-axis and line y = mx
52
(where m > 0). If product of radii of the circles S1 = 0 and S2 = 0 is , then find the value of m.
3

Get 10% Instant Discount On Unacademy Plus [Use Referral Code: MCSIRLIVE] 33
Maths IIT-JEE ‘Best Approach’ (MC SIR) Circle

EXERCISE–II
Q.1 Show that the equation of a straight line meeting the circle x2 + y2 = a2 in two points at equal distances

'd' from a point (x1 , y1) on its circumference is xx1 + yy1  a2 + d 2 2 = 0. 
Q.2 A rhombus ABCD has sides of length 10. A circle with centre 'A' passes through C (the opposite vertex)
likewise, a circle with centre B passes through D. If the two circles are tangent to each other, find the
area of the rhombus.
Q.3 Let A, B, C be real numbers such that
(i) (sin A, cos B) lies on a unit circle centred at origin.
(ii) tan C and cot C are defined.
If the minimum value of (tan C – sin A)2 + (cot C – cos B)2 is a + b 2 where a, b  I, find the value
of a3 + b3.
Q.4 An isosceles right angled triangle whose sides are 1, 1, 2 lies entirely in the first quadrant with the
ends of the hypotenuse on the coordinate axes. If it slides prove that the locus of its centroid is
32
(3x  y)2 + (x  3y)2 = .
9 4 y
Q.5 Real number x, y satisfies x2 + y2 = 1. If the maximum and minimum value of the expression z 
are M and m respectively, then find the value (2M + 6m). 7x
Q.6 The radical axis of the circles x2 + y2 + 2gx + 2fy + c = 0 and 2x2 + 2y2 + 3x + 8y + 2c = 0 touches
the circle x² + y² + 2x  2y + 1 = 0. Show that either g = 3/4 or f = 2.
Q.7 Find the equation of the circle through the points of intersection of circles x2 + y2  4x  6y  12=0
and x2 + y2 + 6x + 4y  12 = 0 & cutting the circle x2 + y2  2x  4 = 0 orthogonally.
Q.8 The centre of the circle S = 0 lie on the line 2x  2y + 9 = 0 & S = 0 cuts orthogonally the circle
x2 + y2 = 4. Show that circle S = 0 passes through two fixed points & find their coordinates.
Q.9(a) Find the equation of a circle passing through the origin if the line pair, xy – 3x + 2y – 6 = 0 is orthogonal
to it. If this circle is orthogonal to the circle x2 + y2 – kx + 2ky – 8=0 then find the value of k.
(b) Find the equation of the circle which cuts the circle x2 + y2 – 14x – 8y + 64 = 0 and the coordinate axes orthogonally.
Q.10 Show that the locus of the centres of a circle which cuts two given circles orthogonally is a straight line
& hence deduce the locus of the centers of the circles which cut the circles x2 + y2 + 4x  6y + 9=0 &
x2 + y2  5x + 4y + 2 = 0 orthogonally. Interpret the locus.
Q.11 Find the equation of a circle which touches the line x + y = 5 at the point (2, 7) and cuts the circle
x2 + y2 + 4x  6y + 9 = 0 orthogonally.
Q.12 Find the equation of the circle passing through the point (–6 , 0) if the power of the point (1, 1) w.r.t. the
circle is 5 and it cuts the circle x2 + y2 – 4x – 6y – 3 = 0 orthogonally.
Q.13 Consider a family of circles passing through two fixed points A (3, 7) & B(6, 5). Then the chords in
which the circle x2 + y2 – 4x – 6y – 3 = 0 cuts the members of the family are concurrent at a point.
Find the coordinates of this point.
Q.14 Find the equation of circle passing through (1, 1) belonging to the system of coaxial circles that are
tangent at (2, 2) to the locus of the point of intersection of mutually perpendicular tangent to the circle
x2 + y2 = 4.
Q.15 The circle C : x2 + y2 + kx + (1 + k)y – (k + 1) = 0 passes through two fixed points for every real number
k. Find
(i) the coordinates of these two points. (ii) the minimum value of the radius of a circle C.

Get 10% Instant Discount On Unacademy Plus [Use Referral Code: MCSIRLIVE] 34
Maths IIT-JEE ‘Best Approach’ (MC SIR) Circle
Q.16 Find the equation of a circle which is co-axial with circles 2x2 + 2y2  2x + 6y  3 = 0 &
x2 + y2 + 4x + 2y + 1 = 0. It is given that the centre of the circle to be determined lies on the radical axis
of these two circles.
Q.17 The circles, which cut the family of circles passing through the fixed points A  (2, 1) and B  (4, 3)
orthogonally, pass through two fixed points (x1 , y1) and (x2 , y2), which may be real or imaginary. Find
 
the value of x13  x 32  y13  y 32 .
Q.18 Find the equation of a circle which touches the lines 7x2 – 18xy + 7y2 = 0 and the circle
x2 + y2 – 8x – 8y = 0 and is contained in the given circle.
Q.19 Find the equation of the circle which passes through the origin, meets the x-axis orthogonally & cuts the
circle x2 + y2 = a2 at an angle of 45º.
Q.20 Consider two circles C1 of radius 'a' and C2 of radius 'b' (b > a) both lying in the first quadrant and
touching the coordinate axes. In each of the conditions listed in column-I, the ratio of b a is given in
column-II.
Column-I Column-II
(A) C1 and C2 touch each other (P) 2 2
(B) C1 and C2 are orthogonal (Q) 3
(C) C1 and C2 intersect so that the common chord is longest (R) 2 3
(D) C2 passes through the centre of C1 (S) 32 2
(T) 32 2
EXERCISE–III
Q. 1 (a) If two distinct chords, drawn from the point (p, q) on the circle x2 + y2 = px + qy (where
pq  q) are bisected by the x-axis, then
(A) p2 = q2 (B) p2 = 8q2 (C) p2 < 8q2 (D) p2 > 8q2

(b) Let L, be a striaght line through the origin and L2 be the straight line x + y = 1. If the intercepts
made by the circle x2 + y2 – x + 3y = 0, on L1 & L2 are equal, then which of the following
equations can represent L1 ?
(A) x + y = 0 (B) x – y = 0 (C) x + 7y = 0 (D) x – 7y = 0
(c) Let T1 , T2 be two tangents drawn from (–2, 0) onto the circle C : x2 + y2 = 1. Determine the
circles touching C and having T1 , T2 as their pair of tangents. Further, find the equations of all
possible common tangents to these circles, when taken two at a time.
[JEE '99, 2+3+10]
Q.2 (a) 2 2
The triangle PQR is inscribed in the circle, x + y = 25. If Q and R have co-ordinates (3, 4) &
( 4, 3) respectively, then  QPR is equal to
(A) /2 (B) /3 (C) /4 (D) /6
(b) If the circles, x2 + y2 + 2 x + 2 k y + 6 = 0 & x2 + y2 + 2 k y + k = 0 intersect orthogonally,
then ' k ' is :
(A) 2 or  3/2 (B)  2 or  3/2 (C) 2 or 3/2 (D)  2 or 3/2
[JEE '2000 (Screening), 1+1]
Q.3 (a) Extremities of a diagonal of a rectangle are (0, 0) & (4, 3). Find the equation of the tangents to
the circumcircle of a rectangle which are parallel to this diagonal.
(b) Find the point on the straight line, y = 2 x + 11 which is nearest to the circle,
16(x2 + y2) + 32x  8y  50 = 0.

Get 10% Instant Discount On Unacademy Plus [Use Referral Code: MCSIRLIVE] 35
Maths IIT-JEE ‘Best Approach’ (MC SIR) Circle
(c) A circle of radius 2 units rolls on the outerside of the circle, x2 + y2 + 4 x = 0 , touching it
externally. Find the locus of the centre of this outer circle. Also find the equations of the common
tangents of the two circles when the line joining the centres of the two circles is inclined at an
angle of 60º with x-axis. [REE '2000 (Mains) 3 + 3 + 5]
Q.4 (a) Let PQ and RS be tangents at the extremities of the diameter PR of a circle of radius r. If PS and
RQ intersect at a point X on the circumference of the circle then 2r equals
2 2

(A) PQ  RS (B)
PQ  RS
(C)
2PQ  RS
(D)
 PQ   RS
2 PQ  RS 2
[ JEE '2001 (Screening) 1 out of 35]
(b) Let 2x2 + y2 – 3xy = 0 be the equation of a pair of tangents drawn from the origin 'O' to a circle
of radius 3 with centre in the first quadrant. IfA is one of the points of contact, find the length of OA.
[JEE '2001 (Mains) 5 out of 100]
Q.5 (a) Find the equation of the circle which passes through the points of intersection of circles
x2 + y2 – 2x – 6y + 6 = 0 and x2 + y2 + 2x – 6y + 6 = 0 and intersects the circle
x2 + y2 + 4x + 6y + 4 = 0 orthogonally. [REE '2001 (Mains) 3 out of 100]
(b) Tangents TP and TQ are drawn from a point T to the circle x2 + y2 = a2. If the point T lies on the
line px + qy = r, find the locus of centre of the circumcircle of triangle TPQ.
[ REE '2001 (Mains) 5 out of 100 ]
Q.6 (a) If the tangent at the point P on the circle x2 + y2 + 6x + 6y = 2 meets the straight line
5x – 2y + 6 = 0 at a point Q on the y-axis, then the length of PQ is
(A) 4 (B) 2 5 (C) 5 (D) 3 5
(b) 2
If a > 2b > 0 then the positive value of m for which y = mx – b 1 m is a common tangent to
x2 + y2 = b2 and (x – a)2 + y2 = b2 is
2b a 2  4b 2 2b b
(A) 2 2 (B) (C) (D)
a  4b 2b a  2b a  2b
[ JEE '2002 (Scr)3 + 3 out of 270]
Q.7 The radius of the circle, having centre at (2, 1), whose one of the chord is a diameter of the circle
x2 + y2 – 2x – 6y + 6 = 0
(A) 1 (B) 2 (C) 3 (D) 3 [JEE '2004 (Scr)]
Q.8 Line 2x + 3y + 1 = 0 is a tangent to a circle at (1, -1). This circle is orthogonal to a circle which is drawn
having diameter as a line segment with end points (0, –1) and (– 2, 3). Find equation of circle.
[JEE '2004, 4 out of 60]
Q.9 A circle is given by x2 + (y – 1)2 = 1, another circle C touches it externally and also the x-axis, then the
locus of its centre is
(A) {(x, y) : x2 = 4y}  {(x, y) : y  0} (B) {(x, y) : x2 + (y – 1)2 = 4}  {x, y) : y  0}
(C) {(x, y) : x2 = y}  {(0, y) : y  0} (D) {(x, y) : x2 = 4y}  {(0, y) : y  0}
[JEE '2005 (Scr)]
Q.10(a) Let ABCD be a quadrilateral with area 18, with side AB parallel to the side CD and AB = 2CD. Let AD
be perpendicular to AB and CD. If a circle is drawn inside the quadrilateral ABCD touching all the sides,
then its radius is
(A) 3 (B) 2 (C) 3/2 (D) 1
2 2
(b) Tangents are drawn from the point (17, 7) to the circle x + y = 169.
Statement-1: The tangents are mutually perpendicular.
because
Statement-2: The locus of the points from which mutually perpendicular tangents can be drawn to the
given circle is x2 + y2 = 338.

Get 10% Instant Discount On Unacademy Plus [Use Referral Code: MCSIRLIVE] 36
Maths IIT-JEE ‘Best Approach’ (MC SIR) Circle
(A) Statement-1 is true, statement-2 is true; statement-2 is correct explanation for statement-1.
(B) Statement-1 is true, statement-2 is true; statement-2 is NOT a correct explanation for statement-1.
(C) Statement-1 is true, statement-2 is false.
(D) Statement-1 is false, statement-2 is true. [JEE 2007, 3+3]
Q.11(a) Consider the two curves
C1 : y2 = 4x ; C2 : x2 + y2 – 6x + 1 = 0. Then,
(A) C1 and C2 touch each other only at one point
(B) C1 and C2 touch each other exactly at two points
(C) C1 and C2 intersect (but do not touch) at exactly two points
(D) C1 and C2 neither intersect nor touch each other
(b) Consider, L1 : 2x + 3y + p – 3 = 0 ; L2 : 2x + 3y + p + 3 = 0,
where p is a real number, and C : x2 + y2 + 6x – 10y + 30 = 0.
STATEMENT-1 : If line L1 is a chord of circle C, then line L2 is not always a diameter of circle C.
and
STATEMENT-2 : If line L1 is a diameter of circle C, then line L2 is not a chord of circle C.
(A) Statement-1 is True, Statement-2 is True; statement-2 is a correct explanation for statement-1
(B) Statement-1 is True, Statement-2 is True; statement-2 is NOT a correct explanation for statement-1
(C) Statement-1 is True, Statement-2 is False
(D) Statement-1 is False, Statement-2 is True
(c) Comprehension (3 questions together):
A circle C of radius 1 is inscribed in an equilateral triangle PQR. The points of contact of C with the sides
PQ, QR, RP are D, E, F respectively. The line PQ is given by the equation 3 x + y – 6 = 0 and the
3 3 3
point D is  ,  . Further, it is given that the origin and the centre of C are on the same side of the
 2 2
line PQ.
(i) The equation of circle C is
1
(A) (x – 2 3 )2 + (y – 1)2 = 1 (B) (x – 2 3 )2 + (y + )2 = 1
2
(C) (x – 3 )2 + (y + 1)2 = 1 (D) (x – 3 )2 + (y – 1)2 = 1
(ii) Points E and F are given by
 3 3  3 1
(A)  2 , 2  ,
 
 3 ,0  (B)  2 , 2  ,
 
 3 ,0 
 3 3  3 1 3 3  3 1
(C)  2 , 2  ,  2 , 2  (D)  2 , 2  ,  
 2 , 2
       

(iii) Equations of the sides RP, RQ are


2 2 1
(A) y = x + 1, y = – x–1 (B) y = x, y = 0
3 3 3

3 3
(C) y = x + 1, y = – x–1 (D) y = 3 x, y = 0
2 2
[JEE 2008, 3+3 + 4 + 4 + 4]

Get 10% Instant Discount On Unacademy Plus [Use Referral Code: MCSIRLIVE] 37
Maths IIT-JEE ‘Best Approach’ (MC SIR) Circle
Q.12 Tangents drawn from the point P(l, 8) to the circle x2 + y2 – 6x – 4y – 11 = 0 [IIT 2009]
touch the circle at the points A and B. The equation of the circumcircle of the triangle PAB is
(A) x2 + y2 + 4x – 6x + 19 = 0 (B) x2 + y2 – 10y + 19 = 0
(C) x2 + y2 – 2x + 6y – 29 = 0 (D) x2 + y2 – 6x – 4y + 19 = 0
Q. 13 The centres of two circles C1 and C2 each of unit radius are at a distance of 6 units from each other. Let
P be the mid point of the line segment joining the centres of C1 and C2 and C be a circle touching circles
C1 and C2 externally. If a common tangent to C1 and C passing through P is also a common tangent to
C2 and C, then the radius of the circle C is [JEE 2009, 3 + 4]

Q. 14 The circle passing through the point (–1, 0) and touching the y-axis at (0, 2) also passes through the point
 3   5   3 5
(A)   , 0  (B)   , 2  (C)   ,  (D) (– 4, 0) [IIT 2011]
 2   2   2 2

Q. 15 The straight line 2x – 3y = 1 divides the circular region x2 + y2 6 into two parts. If
 3   5 3   1 1   1 1  
S =  2,  ,  ,  ,  ,   ,  ,   [IIT 2011]
 4   4 4   4 4   8 4  
then the number of point(s) in S lying inside the smaller part is
Q. 16 The locus of the mid-point of the chord of contact of tangents drawn from points lying on the straight line
4x – 5y = 20 to the circle x2 + y2 = 9 is [IIT 2012]
2 2
(A) 20(x + y ) – 36x + 45y = 0 2 2
(B) 20(x + y ) + 36x – 45y = 0
2 2
(C) 36(x + y ) – 20x + 45y = 0 (D) 36(x2 + y2) + 20x – 45y = 0

Paragraph for Question No. 17 to 18


A tangent PT is drawn to the circle x2 + y2 = 4 at the point P ( 3,1) . A straight line L, perpendicular to
PT is a tangent to the circle (x – 3)2 + y2 = 1 [IIT 2012]

Q. 17 A common tangent of the two circles is


(A) x = 4 (B) y = 2 (C) x + 3 y = – 1 (D) x + 2 2 y = 6

Q. 18 A possible equation of L is
(A) x – 3 y = + 1 (B) x + 3 y = 1 (C) x – 3 y = – 1 (D) x + 3 y = 5

Q. 19 The circle passing through (1, – 2) and touching the axis of x at (3, 0) also passes through the point :
(A) (–2, 5) (B) (–5, 2) (C) (2, –5) (D) (5, –2)
[IIT JEE Main 2013]

Q. 20 Circle(s) touching x-axis at a distance 3 from the origin and having an intercept of length 2 7 on y-axis
is (are) [IIT JEE Adv. 2013]
2 2
(A) x + y – 6x + 8y + 9 = 0 2 2
(B) x + y – 6x + 7y + 9 = 0
2 2
(C) x + y – 6x – 8y + 9 = 0 (D) x2 + y2 – 6x – 7y + 9 = 0

Q. 21 Let C be the circle with centre at (1, 1) and radius = 1. If T is the circle centred at (0, y), passing through
origin and touching the circle C externally, then the radius of T is equal to [IIT JEE Main 2014]

1 3 3 1
(A) (B) (C) (D)
4 2 2 2
Get 10% Instant Discount On Unacademy Plus [Use Referral Code: MCSIRLIVE] 38
Maths IIT-JEE ‘Best Approach’ (MC SIR) Circle
Q. 22 A circle S passes through the point (0, 1) and is orthogonal to the circles (x – 1) 2 + y2 = 16 and
x2 + y2 = 1. Then, [IIT JEE Adv. 2014]
(A) radius of S is 8 (B) radius of S is 7
(C) centre of S is (–7, 1) (D) centre of S is (–8, 1)

Q. 23 The number of common tangents to the circles x2 + y2 – 4x – 6y – 12 = 0 and x2 + y2 + 6x + 18y + 26 = 0


is : [IIT JEE Main 2015]
(A) 4 (B) 1 (C) 2 (D) 3

Q. 24 Locus of the image of the point (2, 3) in the line (2x – 3y + 4) + k(x – 2y + 3) = 0, k  R, is a
(A) Circle of radius 3 (B) straight line parallel to x-axis
(C) straight line parallel to y-axis (D) circle of radius 2 [IIT JEE Main 2015]

Q. 25 The centres of those circles which touch the circle, x2 – y2 – 8x – 8y – 4 = 0, externally and also touch
the x-axis, lie on : [IIT JEE Main 2016]
(A) a circle (B) an ellipse which is not a circle
(C) a hyperbola (D) a parabola
Q. 26 If one of the diameters of the circle, given by the equation, x2 + y2 – 4x + 6y – 12 = 0, is a chord of a
circle S, whose centre is at (–3, 2), then the radius of S is : [IIT JEE Main 2016]
(A) 5 2 (B) 5 3 (C) 5 (D) 10

Q. 27 Let P be the point on the parabola, y2 = 8x which is at a minimum distance from the centre C of the circle,
x2 + (y + 6)2 = 1. Then the equation of the circle, passing through C and having its centre at P is
(A) x2 + y2 – 4x + 8y + 12 = 0 (B) x2 + y2 – x + 4y – 12 = 0 [IIT JEE Main 2016]
x
(C) x2 + y2 – + 2y – 24 = 0 (D) x2 + y2 – 4x + 9y + 18 = 0
4

Q. 28 The circle C1 : x2 + y2 = 3, with centre at O, intersects the parabola x2 = 2y at the point P in the first
quadrant. Let the tangent to the circle C1 at P touches other two circles C2 and C3 at R2 and R3,
respectively. Suppose C2 and C3 have equal radii 2 3 and centres Q2 and Q3, respectively. If Q2 and
Q3 lie on the y-axis, then [IIT JEE Adv. 2016]
(A) Q2Q3 = 12 (B) R2R3 = 4 6
(C) area of the triangle OR2R3 is 6 2 (D) area of the triangle PQ2Q3 is 4 2

Q. 29 Let RS be the diameter of the circle x2 + y2 = 1, where S is the point (1, 0). Let P be a variable point
(other than R and S) on the circle and tangents to the cricle at S and P meet at the point Q. The normal
to the circle at P intersects a line drawn through Q parallel to RS at point E. Then the locus of E passes
through the point(s). [IIT JEE Adv. 2016]
1 1  1 1 1 1  1 1
(A)  ,  (B)  ,  (C)  ,   (D)  ,  
3 3 4 2 3 3 4 2
Q. 30 Let  R. Consider the system of linear equations
x + 2y = 
3x – 2y = 

Get 10% Instant Discount On Unacademy Plus [Use Referral Code: MCSIRLIVE] 39
Maths IIT-JEE ‘Best Approach’ (MC SIR) Circle
Which of the following statement(s) is(are) correct ? [IIT JEE Adv. 2016]
(A) If  = –3, then the system has infinitely many solutions for all values of  and 
(B) If  –3, then the system has a unique solution for all values of and .
(C) If  +  = 0, then the system has infinitely many solutions for  = –3
(D) If  +  0, then the system has no solution for  = –3

Q. 31 For how many values of p, the circle x2 + y2 + 2x + 4y – p = 0 and the coordinate axes have exactly three
common points ? [JEE Adv. 2017]

Paragraph "X"
Let S be the circle in the xy-plane defined by the equation x2 + y2 = 4. [JEE Adv. 2018]
(There are two question based on Paragraph "X", the question given below is one of them)

Q. 32 Let E1E2 and F1F2 be the chord of S passing through the point P0(1, 1) and parallel to the x-axis and the
y-axis, respectively. Let G1G2 be the chord of S passing through P0 and having slope –1. Let the tangents
to S at E1 and E2 meet at E3, the tangents of S at F1 and F2 meet at F3, and the tangents to S at G1 and
G2 meet at G3. Then, the points E3, F3 and G3 lie on the curve
(A) x + y = 4 (B) (x – 4)2 + (y – 4)2 = 16
(C) (x – 4) (y – 4) = 4 (D) xy = 4

Q. 33 Let P be a point on the circle S with both coordinates being positive. Let the tangent to S at P intersect
the coordinate axes at the points M and N. Then, the mid-point of the line segment MN must lie on the
curve -
(A) (x + y)2 = 3xy (B) x2/3 + y2/3 = 24/3
(C) x2 + y2 = 2xy (D) x2 + y2 = x2y2

Q. 34 Let T be the line passing through the points P(–2, 7) and Q(2, –5). Let F1 be the set of all pairs of circles
(S1, S2) such that T is tangents to S1 at P and tangent to S2 at Q, and also such that S1 and S2 touch each
other at a point, say, M. Let E1 be the set representing the locus of M as the pair (S1, S2) varies in F1.
Let the set of all straight line segments joining a pair of distinct points of E1 and passing through the point
R(1, 1) be F2. Let E2 be the set of the mid-points of the line segments in the set F2. Then, which of the
following statement(s) is (are) TRUE ? [JEE Adv. 2018]
4 7
(A) The point (–2, 7) lies in E1 (B) The point  ,  does NOT lie in E2
5 5
1  3
(C) The point  ,1 lies in E2 (D) The point  0,  does NOT lie in E1
2   2

Q.35 Three circles of radii a,b,c (a < b < c) touch each other externally. If they have x - axis as a common
tangent, then : [Jee main 2019 (09-01-2019-shift-1)]
(A) a,b,c are in A.P (B) a , b, c are In A.P..
1 1 1 1 1 1
(C) = + (D) = +
b a c a b c

Get 10% Instant Discount On Unacademy Plus [Use Referral Code: MCSIRLIVE] 40
Maths IIT-JEE ‘Best Approach’ (MC SIR) Circle
Q.36 If the circles x2 + y2 – 16x – 20y + 164 = r2 and (x – 4)2 + (y – 7)2 = 36 intersect at two distinct points,
then; [Jee main 2019 (09-01-2019-shift-2)]
(A) 0 < r < 1 (B) r = 11 (C) r > 11 (D) 1 < r < 11

Q.37 If a circle C passing through the point (4,0) touches the circle x2 + y2 + 4x – 6y = 12 externally at the
point (1, –1), then the radius of C is : [Jee main 2019 (10-01-2019-shift-1)]
(A) 57 (B) 5 (C) 2 5 (D) 4

Q.38 If the area of an equilateral triangle inscribed in the circle, x2 + y2 +10x + 12y + c = 0 is 27 3 sq. units
then c is equal to: [Jee main 2019 (10-01-2019-shift-2)]
(A) 13 (B) – 25 (C) 25 (D) 20

Q.39 Two circles with equal radii are intersecting at the points (0,1) and (0,–1).The tangent at the point (0,1)
to one of the circles passes through the centre of the other circle. Then the distance between the centres
of these circles is : [Jee main 2019 (11-01-2019-shift-1)]
(A) 1 (B) 2 (C) 2 (D) 2 2

Q.40 The straight line x + 2y = 1 meets the coordinate axes at A and B. A circle is drawn through A,B and the
origin. Then the sum of perpendicular distances from A and B on the tangent to the circle at the origin
is : [Jee main 2019 (11-01-2019-shift-1)]
5 5
(A) (B) 4 5 (C) (D) 2 5
4 2

Q.41 A square is inscribed in the circle x2 + y2 – 6x + 8y – 103 = 0 with its sides parallel to the coordinates
axes. Then the distance of the vertex of this square which is nearest to the origin is :
[Jee main 2019 (11-01-2019-shift-1)]
(A) 6 (B) 4 5 (C) 137 (D) 13

Q.42 If a variable line, 3x + 4y –  = 0 is such that the two circles x2 + y2 – 2x – 2y + 1 = 0 and


x2 + y2 – 18x – 2y + 78 = 0 are on its opposite sides, then the set of all values of  is the interval :
[Jee main 2019 (12-01-2019-shift-1)]
(A) (23,31) (B) [12,21] (C) (2,17) (D) [13,23]

Q.43 Let C1 and C2 be the centres of the circles x2 + y2 – 2x – 2y – 2 = 0 and x2 + y2 – 6x – 6y + 14 = 0


respectively. If P and Q are the points of intersection of these circles, then the area (in sq. units) of the
quadrilateral PC1 QC2 is : [Jee main 2019 (12-01-2019-shift-1)]
(A) 8 (B) 4 (C) 6 (D) 9
Q.44 If a circle of radius R passes through the origin O and intersects the coordinate axes at A and B, then the
locus of the foot of perpendicular from O on AB is : [Jee main 2019 (12-01-2019-shift-2)]
2 3

(A) x 2 + y 2  = 4R x y
2 2 2

(B) x 2 + y 2  = 4R 2 x 2 y 2
2
(C)  x   x + y  = R xy (D)  x +y 
2
+ y2 2 2 2
= 4Rx 2 y 2

Get 10% Instant Discount On Unacademy Plus [Use Referral Code: MCSIRLIVE] 41
Maths IIT-JEE ‘Best Approach’ (MC SIR) Circle
Q.45 The sum of the squares of the lengths of the chords intercepted on the circle, x2+y2=16, by the lines,
x + y = n, n  N , where N is the set of all natural numbers, is : [Jee main 2019 (08-04-2019-shift-1)]
(A) 105 (B) 160 (C) 210 (D) 320

Q.46 The tangent and the normal lines at the point  


3,1 to the circle x2 + y2 = 4 and the x-axis form a
triangle. The area of this triangle (in square units) is: [Jee main 2019 (08-04-2019-shift-2)]
2 1 1 4
(A) (B) (C) (D)
3 3 3 3

Q.47 If a tangent to the circle x2 + y2 = 1 intersects the coordinate axes at distinct points P and Q, then the
locus of the mid- point of PQ is : [Jee main 2019 (09-04-2019-shift-1)]
2 2 2 2
(A) x + y – 2x y = 0 (B) x + y – 16x2y2 = 0
2 2

2 2
(C) x + y – 2xy = 0 (D) x2 + y2 – 4x2y2 = 0

Q.48 The common tangent to the circles x2 + y2 = 4 and x2 + y2 + 6x + 8y – 24 = 0 also passes through the
point : [Jee main 2019 (09-04-2019-shift-2)]
(A) (– 4, 6) (B) (– 6, 4) (C) (6, – 2) (D) (4, – 2)

Q.49 A rectangle is inscribed in a circle with a diameter lying along the line 3y = x + 7. If the two adjacent
vertices of the rectangle are (–8,5) and (6,5), then the area of the rectangle (in sq. units) is :
[Jee main 2019 (09-04-2019-shift-2)]
(A) 98 (B) 84 (C) 72 (D) 56

Q.50 The line x = y touches a circle at the point (1,1). If the circle also passes through the point (1, – 3), then
its radius is : [Jee main 2019 (10-04-2019-shift-1)]
(A) 3 (B) 2 2 (C) 2 (D) 3 2

Q.51 If the circles x2 + y2 + 5Kx + 2y + K = 0 and 2(x2 + y2) + 2Kx + 3y – 1 = 0, (KR), intersect at the
points P and Q, then the line 4x + 5y – K = 0 passes through P and Q for:
[Jee main 2019 (10-04-2019-shift-1)]
(A) no value of K (B) exactly two values of K
(C) exactly one value of K (D) infinitely many values of K

Q.52 The locus of the centres of the circles, which touch the circle, x2+y2=1 externally, also touch the y-axis
and lie in the first quadrant is, [Jee main 2019 (10-04-2019-shift-2)]
(A) x = 1 + 2y, y  0 (B) y = 1 + 2x, x  0

(C) y = 1 + 4x, x  0 (D) x = 1 + 4y, y  0

Q.53 If the angle of intersection at a point where the two circles with radii 5cm and 12 cm intersect in 900, then
the length (in cm) of their common chord is : [Jee main 2019 (12-04-2019-shift-1)]
13 120 13 60
(A) (B) (C) (D)
5 13 2 13

Get 10% Instant Discount On Unacademy Plus [Use Referral Code: MCSIRLIVE] 42
Maths IIT-JEE ‘Best Approach’ (MC SIR) Circle

Q.54 A circle touching the x - axis at (3,0) and making an intercept of length 8 on the y - axis passes through
the point : [Jee main 2019 (12-04-2019-shift-2)]
(A) (1,5) (B) (2,3) (C) (3,10) (D) (3,5)

Q.55 A line y = mx + 1 intersects the circle (x – 3)2 + (y + 2)2 = 25 at the points P and Q. If the midpoint of
3
the line segment PQ has x-coordinate  , then which one of the following options is correct ?
5
[JEE Advanced 2019]
(A) 6  m < 8 (B) 2  m < 4 (C) 4  m < 6 (D) – 3  m < – 1
Q.56 Let the point B be the reflection of the point A(2, 3) with respect to the line 8x – 6y – 23 = 0. Let A and
B be circles of radii 2 and 1 with centres A and B respectively. Let T be a common tangent to the circles
A and B such that both the circles are on the same side of T. If C is the point of intersection of T and
the line passing through A and B, then the length of the line segment AC is _______
[JEE Advanced 2019]

Q.57 Let the tangents drawn from the origin to the circle, x2 + y2 – 8x – 4y + 16 = 0 touch it at the points
A and B. The (AB)2 is equal to:
56 52
(A) (B)
5 5
64 32
(C) (D)
5 5

Q.58 If a line, y = mx + c is a tangent to the circle, (x – 3)2 + y2 = 1 and it is perpendicular to a line


 1 1 
L1, where L1 is the tangent to the circle, x2 + y2 = 1 at the point  ,  ; then :
 2 2

(A) c2 – 7c + 6 = 0 (B) c2 + 7c + 6 = 0 (C) c2 – 6c + 7 = 0 (D) c2+ 6c + 7 = 0

Q.59 A circle touches the y-axis at the point (0,4) and passes through the point (2,0).
Which of the following lines is not a tangent to this circle ?
(A) 3x – 4y – 24 = 0 (B) 4x – 3y + 17 = 0
(C) 3x + 4y – 6 = 0 (D) 4x +3y – 8 = 0

Q.60 If the curves, x2 – 6x + y2 + 8 = 0 and x2 – 8 y + y2 + 16 – k = 0, ( k > 0) touch each


other at a point, then the largest value of k is _______________.

Q.61 The number of integral values of k for which the line, 3x+4y=k intersects the circle, x2 + y2 – 2x – 4y + 4 = 0
at two distinct points is................ [Jee main 2020 (02-09-2020-shift-1)]

Get 10% Instant Discount On Unacademy Plus [Use Referral Code: MCSIRLIVE] 43
Maths IIT-JEE ‘Best Approach’ (MC SIR) Circle

Q.62 The set of all possible values of  in the interval (0,  ) for which the points (1,2) and  sin , cos   lie
on the same side of the line x + y = 1 is : [Jee main 2020 (02-09-2020-shift-2)]
     3    3 
(A)  0,  (B)  0,  (C)  0,  (D)  , 
 4  2  4  4 4 

Q.63 The diameter of the circle, whose centre lies on the line x + y = 2 in the first quadrant and which touches
both the lines x = 3 and y = 2, is .......... [Jee main 2020 (03-09-2020-shift-1)]

Q.64 The circle passing through the intersection of the circles, x2 + y2 – 6x = 0 and x2 + y2 – 4y = 0, having its
centre on the line, 2x – 3y + 12=0, also passes through the point :
[ Jee main 2020 (04-09-2020-shift-2)]
(A) (–1,3) (B) (1,–3) (C) (–3,6) (D) (–3,1)

Q.65 Let PQ be a diameter of the circle x2 + y2 = 9. If  and  are the lengths of the perpendiculars from P
and Q on the straight line, x + y = 2 respectively, then the maximum value of αβ is _____
[Jee main 2020 (04-09-2020-shift-2)]
Q.66 If the length of the chord of the circle, x + y = r (r > 0) along the line, y – 2x = 3 is r, then r2 is equal to
2 2 2

: [Jee main 2020 (05-09-2020-shift-2)]


24 9 12
(A) 12 (B) (C) (D)
5 5 5
5
Q.67 Let O be the centre of the circle x2 + y2 = r2, where r  . Suppose PQ is a chord of this circle and
2
the equation of the line passing through P and Q is 2x + 4y = 5. If the centre of the circumcircle of the
triangle OPQ lies on the line x + 2y = 4, then the value of r is [JEE Advanced 2020]

Q.68 If one of the diameters of the circle x2 + y2 – 2x – 6y + 6 = 0 is a chord of another

circle ‘C’, whose center is at (2, 1), then its radius is ______ .

Q.69 Let a point P be such that its distance from the point (5, 0) is thrice the distance of P
from the point (– 5, 0). If the locus of the point P is a circle of radius r, then 4r2 is
equal to ________.

Q.70 If the area of the triangle formed by the positive x-axis, the normal and the tangent
to the circle (x – 2)2 + (y – 3)2 = 25 at the point (5, 7) is A, then 24A is equal to_____.

Get 10% Instant Discount On Unacademy Plus [Use Referral Code: MCSIRLIVE] 44
Maths IIT-JEE ‘Best Approach’ (MC SIR) Circle
Q.71 In the circle given below, let OA = 1 unit, OB = 13 unit and PQ  OB. Then, the area of the triangle
PQB (in square units) is :

(A) 24 2 (B) 24 3 (C) 26 3 (D) 26 2

Q.72 If the locus of the mid-point of the line segment from the point (3, 2) to a point on the

circle, x2 + y2 = 1 is a circle of radius r, then r is equal to :

1 1 1
(A) 1 (B) (C) (D)
2 3 4

Q.73 Let A(1, 4) and B(1, –5) be two points. Let P be a point on the circle (x – 1)2 + (y – 1)2 = 1 such that
(PA)2 + (PB)2 have maximum value, then the points P, A and B lie on :

(A) a straight line (B) a hyperbola

(C) an ellipse (D) a parabola

Q.74 Let the normals at all the points on a given curve pass through a fixed point (a, b).

If the curve passes through (3, –3) and (4, 2 2) , and given that a  2 2b  3 ,

then (a2 + b2 + ab) is equal to ______.

Q.75 Let ABCD be a square of side of unit length. Let a circle C1 centered at A with unit
radius is drawn. Another circle C2 which touches C1 and the lines AD and AB are
tangent to it, is also drawn. Let a tangent line from the point C to the circle C2 meet

the side AB at E. If the length of EB is   3 , where  are integers, then  is

equal to_______.

Get 10% Instant Discount On Unacademy Plus [Use Referral Code: MCSIRLIVE] 45
Maths IIT-JEE ‘Best Approach’ (MC SIR) Circle

Q.76 Let the lengths of intercepts on x-axis and y-axis made by the circle x2 + y2 + ax + 2ay + c = 0, (a < 0)
be 2 2 and 2 5 , respectively. Then the shortest distance from origin to a tangent to this circle which
is perpendicular to the line x + 2y = 0, is equal to :

(A) 11 (B) 7

(C) 6 (D) 10

Q.77 The line 2x – y + 1 = 0 is a tangent to the circle at the point (2, 5) and the centre of the
circle lies on x – 2y = 4. Then, the radius of the circle is:

(A) 3 5 (B) 5 3 (C) 5 4 (D) 4 5

Q.78 Choose the incorrect statement about the two circles whose equations are given below :

x2 + y2 – 10x – 10y + 41 = 0 and

x2 + y2 – 16x – 10y + 80 = 0

(A) Distance between two centres is the average of radii of both the circles.

(B) Both circles centres lie inside region of one another.

(C) Both circles pass through the centre of each other.

(D) Circles have two intersection points.


Q.79 The minimum distance between any two points P1 and P2 while considering point P1 on one circle and
point P2 on the other circle for the given circles’ equations
x2 + y2 –10x – 10y + 41 = 0
x2 + y2 – 24x – 10y + 160 = 0 is ______ .

Q.80 Two tangents are drawn from a point P to the circle x2 + y2 – 2x – 4y + 4 = 0, such

 12   12 
that the angle between these tangents is tan 1   , where tan 1    (0, ).
 5  5

If the centre of the circle is denoted by C and these tangents touch the circle at
points A and B, then the ratio of the areas of PAB and CAB is :

(A) 11 : 4 (B) 9 : 4 (C) 3 :1 (D) 2 : 1

Get 10% Instant Discount On Unacademy Plus [Use Referral Code: MCSIRLIVE] 46
Maths IIT-JEE ‘Best Approach’ (MC SIR) Circle

Q.81 Let the tangent to the circle x2 + y2 = 25 at the point R(3, 4) meet x-axis and y-axis at point P and Q,
respectively. If r is the radius of the circle passing through the origin O and having centre at the incentre
of the triangle OPQ, then r2 is equal to

529 125 625 585


(A) (B) (C) (D)
64 72 72 66

Q.82 Choose the correct statement about two circles whose equations are given below :

x2 + y2 – 10x – 10y + 41 = 0

x2 + y2 – 22x – 10y + 137 = 0

(A) circles have same centre (B) circles have no meeting point

(C) circles have only one meeting point (D) circles have two meeting points

Q.83 For the four circles M, N, O and P, following four equations are given :
Circle M : x2 + y2 = 1
Circle N : x2 + y2 – 2x = 0
Circle O : x2 + y2 – 2x – 2y + 1 = 0
Circle P : x2 + y2 – 2y = 0
If the centre of circle M is joined with centre of the circle N, further centre of circle N is joined with
centre of the circle O, centre of circle O is joined with the centre of circle P and lastly, centre of circle P
is joined with centre of circle M, then these lines form the sides of a :
(A) Rhombus (B) Square (C) Rectangle (D) Parallelogram

Q.84 Let S1 : x2 + y2 = 9 and S2 : (x – 2)2 + y2 = 1. Then the locus of center of a variable


circle S which touches S1 internally and S2 externally always passes through the points :

1 5  3
(A) (0,  3) 
(B)  2 ,   (C)  2,   (D) (1, ± 2)
 2   2

Q.85 Let r1 and r2 be the radii of the largest and smallest circles, respectively, which pass

through the point (–4, 1) and having their centres on the circumference of the circle

r1
x2 + y2 + 2x + 4y – 4 = 0. If r  a  b 2 , then a + b is equal to :
2

(A) 7 (B) 11 (C) 5 (D) 3

Get 10% Instant Discount On Unacademy Plus [Use Referral Code: MCSIRLIVE] 47
Maths IIT-JEE ‘Best Approach’ (MC SIR) Circle

Q.86 Let the circle S: 36x2 + 36y2 – 108x + 120y + C = 0 be such that it neither intersects nor touches the co-
ordinates axes. If the point of intersection of the lines, x – 2y = 4 and 2x – y = 5 lies inside the circle S,
then:

25 13
(A) C (B) 100 < C < 165
9 3

(C) 100 < C < 156 (D) 81 < C < 156

Q.87 Let

A = {(x, y)  R × R | 2x2 + 2y2 – 2x – 2y = 1},

B = {(x, y)  R × R | 4x2 + 4y2 – 16y + 7 = 0} and

C = {(x, y)  R × R | x2 + y2 – 4x – 2y + 5 d” r2}.

Then the minimum value of |r| such that A  B  C is equal to :

3  10 2  10 3 2 5
(A) (B) (C) (D) 1  5
2 2 2

Q.88 Let P and Q be two distinct points on a circle which has centre at C(2, 3) and which

passes through origin O. If OC is perpendicular to both the line segments CP and CQ,

then the set {P, Q} is equal to :

(A) {(–1, 5), (5, 1)}

(B) {(2 + 2 2 , 3 – 5 ), (2 – 2 2 , 3 + 5 )}

(C) {(2 + 2 2 , 3 + 5 ), (2 – 2 2 , 3 – 5 )}

(D) {(4, 0), (0, 6)}

Q.89 Two tangents are drawn from the point P(–1, 1) to the circle x2 + y2 – 2x – 6y + 6 = 0. If these tangents
touch the circle at point A and B, and if D is a point on the circle such that length of the segments AB and
AD are equal, then the area of the triangle ABD is equal to:

(A) 4 (B) 2

(C) 3( 2  1) (D) 3( 2  2)

Q.90 Consider a circle C which touches the y-axis at (0, 6) and cuts off intercept 6 5 on the x-axis. Then the
radius of the circle C is equal to :

(A) 8 (B) 53 (C) 9 (D) 82

Get 10% Instant Discount On Unacademy Plus [Use Referral Code: MCSIRLIVE] 48
Maths IIT-JEE ‘Best Approach’ (MC SIR) Circle

Q.91 The locus of a point, which moves such that the sum of squares of its distance from

the points (0,0), (1,0), (0,1), (1,1) is 18 units, is a circle of diameter d. Then d2 is

equal to ______.

Q.92 A circle C touches the line x = 2y at the point (2, 1) and intersects the circle C1 : x 2  y 2  2y  5  0 at
two points P and Q such that PQ is a diameter of C1 . Then the diameter of C is:

(A) 4 15 (B) 7 5 (C) 15 (D) 245

Q.93 Let the equation x 2  y 2  px  11  p  y  5  0 represent circles of varying radius r   0, 5 . Then
the number of elements in the set S  {q : q  p 2 and q is an integer} is ______.

[Jee main 2021 (27-08-2021-shift-1)]


Q.94 Let Z be the set of all integers,

A  x, y   Z  Z;  x  2   y  4
2 2

B   x, y   Z  Z; x 2  y 2  4 and

C  x, y   Z  Z;  x  2  2 2

  y  2  4 [Jee main 2021 (27-08-2021-shift-2)]

If the total number of relations from A  B to A  C is 2p , then the value of p is:

(A) 25 (B) 9 (C) 16 (D) 49


Q.95 Two circles each of radius 5 units touch each other at the point (1, 2). If the equation

of their common tangents is 4x + 3y = 10, and C1  ,   and C2  ,   , C1  C2 are

their centres, then          is equal to ______.

2 2
Q.96 If the variable line 3x + 4y =  lies between the two circles  x  1   y  1  1

2 2
and  x  9    y  1  4 , without intercepting a chord on either circle, then the

sum of all the integral values of  is ______.

Q.97 Let B is the centre of the circle x 2  y 2  2x  4y  1  0 . Let the tangents at two points P and Q on the
 
circle intersect at the point A(3, 1). Then 8   Area APQ  is equal to
 Area BPQ 

Get 10% Instant Discount On Unacademy Plus [Use Referral Code: MCSIRLIVE] 49
Maths IIT-JEE ‘Best Approach’ (MC SIR) Circle
Q.98 Consider a triangle  whose two sides lie on the x-axis and the line x + y + 1 = 0. If the orthocenter of
 is (1, 1), then the equation of the circle passing through the vertices of the triangle is
[Jee Advanced 2021 (Paper-1)]

(A) x2 + y2 – 3x + y = 0 (B) x2 + y2 + x + 3y = 0
(C) x2 + y2 + 2y – 1 = 0 (D) x2 + y2 + x + y = 0

Paragraph

Let

M  (x, y)     :x 2  y 2  r 2 }

1
where r > 0. Consider the geometric progression a n  , n  1, 2,3,...... Let S0 = 0 and, for n 1, let
2 n 1
Sn denote the sum of the first n terms of this progression. For n  1, let Cn denote the circle with center

(Sn–1, 0) and radius an, and Dn denote the circle with center (Sn–1, S n–1) and radius an.
[Jee Advance 2021 (Paper 2)]

1025
Q.99 Consider M with r  . Let k be the number of all those circles Cn that are inside M . L et l be
513
the maximum possible number of circles among these k circles such that no two circles intersect. Then

[Jee Advanced 2021 (Paper-2)]

(A) k + 2l = 22 (B) 2k + l = 26

(C) 2k + 3l = 34 (D) 3k + 2l = 40

(2199  1) 2
Q.100 Consider M with r  . The number of all those circles Dn that are inside M is
2198

[Jee Advanced 2021 (Paper-2)]

(A) 198 (B) 199 (C) 200 (D) 201

Get 10% Instant Discount On Unacademy Plus [Use Referral Code: MCSIRLIVE] 50
Maths IIT-JEE ‘Best Approach’ (MC SIR) Circle
ANSWER KEY
ELEMENTARY EXERCISES

( SL. LONEY - EX-17)


Q.1 x2 + y2 + 2x – 4y = 4 Q.2 x2 + y2 + 10x + 12y = 39
Q.3 x2 + y2 – 2ax + 2by = 2ab Q.4 x2 + y2 + 2ax + 2by + 2b2 = 0

5  1
Q.5 (2, 4); 61 Q.6  ,1 ; 6 13
6 

k  5k
Q.7  ,0 ; Q.8 (g, –f); f 2  g 2
2  2

 c mc 
Q.9  ,  ; |c| Q.13 15x2 + 15y2 – 94x + 18y + 55 = 0
2
 1 m 1  m2 
Q.14 b(x2 + y2 – a2) = x(b2 + h2 – a2) Q.15 x2 + y2 – ax – by = 0
Q.16 x2 + y2 – 22x – 4y + 25 = 0 Q.17 x2 + y2 – 5x – y + 4 = 0
Q.18 3x2 + 3y2 – 29x – 19y + 56 = 0 Q.19 b(x2 + y2) – (a2 + b2)x + (a – b) (a2 + b2) = 0

2 a 2  b2
2
a 2  b2
Q.21 x2 + y2 – 3x – 4y = 0 Q.22 x y  (x  y)  0 ; a  b
ab

Q.23 x2 + y2 – hx – ky = 0 Q.24 x 2  y 2  2 a 2  b 2 y – b2  0
Q.25 x2 + y2 – 10x – 10y + 25 = 0 Q.26 x2 + y2 – 2ax – 2ay + a2 = 0
Q.27 x 2  y 2  2(5  12)(x  y)  37  10 12  0 Q.28 x2 + y2 – 6x + 4y + 9 = 0

Q.29 b(x2 + y2) = x(b2 + c2) Q.30 x 2  y 2  6 2y  6x  9  0

Q.31 x2 + y2 – 3x + 2 = 0; 2x 2  2y 2  5x  3y  3  0 ; 2x 2  2y 2  7x  3y  6  0
Q.33 (x + 21)2 + (y + 13)2 = 652 Q.34 8x2 + 8y2 – 25x – 3y + 18 = 0
Q.36 x2 + y2 = a2 + b2; x2 + y2 – 2(a + b) x + 2 (a – b)y + a2 + b2 = 0

( SL. LONEY - EX-18)


Q.1 5x – 12y = 152 Q.2 24x + 10y + 151 = 0

Q.3 x  2y  2 5 Q.4 x  2y  g  2f   5 g 2  f 2  c

 c c 
Q.5  ,  Q.6 c = a; (0, b)
 2 2
Q.7 Yes Q.8 k = 40 or – 10

Q.9 a cos 2   b sin 2   a 2  b2 sin 2  Q.10 Aa  Bb  C  c A 2  B2

Get 10% Instant Discount On Unacademy Plus [Use Referral Code: MCSIRLIVE] 51
Maths IIT-JEE ‘Best Approach’ (MC SIR) Circle

Q.11 (1) y  mx  a 1  m 2 ; (2) my  x   a 1  m 2 ; (3) ax  y b 2  a 2  ab ; (4) x  y  a 2

a 2b 2
Q.12 2 r2  Q.13 x 2  y 2  2ax  0 ; x 2  y 2  2ay  0
a 2  b2

381
Q.14 c = b – am; c  b  am  (1  m 2 )(a 2  b 2 ) Q.15 x 2  y 2  6x  8y  0
169

Q.16 x2 + y2 – 2cx – 2cy + c2 = 0, where 2c  a  b  a 2  b 2


Q.17 5x2 + 5y2 – 10x + 30y + 49 = 0 Q.18 x2 + y2 – 2cx – 2cy + c2 = 0
Q.19 (x – r)2 + (y – h)2 = r2 Q.20 x2 + y2 – 2x – 2y = 0

( SL. LONEY - EX-19)


Q.1 x + 2y = 7 Q.2 8x – 2y = 11
Q.3 x=0 Q.4 23x + 5y = 57
Q.5 by – ax = a2 Q.6 (5, 10)

3 3 
Q.7  ,  Q.8 (1, – 2)
 5 10 

1 1
Q.9  ,  Q.10 (–2a, –2b)
 2 3

 18  162 243 


Q.11  6,  Q.12 3y – 2x = 13;   , 
 5   13 13 

Q.13 (2, –1) Q.14 x 2  y2  2a 2

1
Q.18 46 Q.19 9
2

 33  1
Q.20 2a 2  2ab  b2 Q.21  , 2;
 4  4
Q.23 (1) 28x2 + 33xy – 28y2 – 715x – 195y + 4225 = 0
(2) 123x2 – 64xy + 3y2 – 664x + 226y + 763 = 0

EXERCISE–I
Q.1 square of side 2; x2 + y2 = 1; x2 + y2 = 2
Q.2 zero, zero
Q.4 32 sq. unit
Q.5 x  y = 0 ; x + 7y = 0
Q.6 (5, 1) & (–1, 5)
Q.7 4x  3y  25 = 0 OR 3x + 4y  25 = 0

Get 10% Instant Discount On Unacademy Plus [Use Referral Code: MCSIRLIVE] 52
Maths IIT-JEE ‘Best Approach’ (MC SIR) Circle
Q.8 (i) (11, 16), (ii) (11, 8), (iii) (11, 12)
Q.9 x2 + y2 – 2x – 2y + 1 = 0 OR x2 + y2 – 42x + 38y – 39 = 0
Q.10 
(i) 3x – 4y = 21; 4x + 3y = 3; (ii) A(0, 1) and B (–1, – 6); (iii) 90°, 5 2  1 units 
(iv) 25 sq. units, 12.5 sq. units; (v) x2 + y2 + x + 5y – 6, x intercept 5; y intercept 7
Q.11 x2 + y2 – 2x – 2y = 0
Q.12 2x – 2y – 3 = 0
Q.13 a2(x2 + y2) = 4x2y2

Q.14 x2 + y2 = a2 + b2; r = a 2  b2
Q.15 2(x2 + y2) + 6x – 17y – 6 = 0
Q.16 63
Q.17 6
Q.18 x  7y = 2, 7x + y = 14; (x  1)2 + (y  7)2 = 32; (x  3)2 + (y + 7)2 = 32 ;
(x  9)2 + (y  1)2 = 32; (x + 5)2 + (y + 1)2 = 32
Q.20 x+y=2
Q.21 10
Q.22 r = 15
Q.23 19
Q.24 (– 4, 2), x2 + y2 – 2x – 6y – 15 = 0
Q.25 x2 + y2 – 6x + 4y=0 OR x2 + y2 + 2x – 8y + 4=0
Q.26 x2 + y2 + x  6y + 3 = 0
Q.27 5x2 + 5y2 – 8x – 14y – 32 = 0
Q.29 64
Q.30 3

EXERCISE–II
Q.2 75 sq. unit Q.3 19
Q.5 4 Q.7 x2 + y2 + 16x + 14y – 12 = 0
Q.8 ( 4, 4) ; (– 1/2, 1/2) Q.9 (a) x2 + y2 + 4x – 6y = 0; k = 1; (b) x2 + y2 = 64
Q.10 9x  10y + 7 = 0; radical axis Q.11 x2 + y2 + 7x  11y + 38 = 0

 23 
Q.12 x2 + y2 + 6x – 3y = 0 Q.13  2, 
 3 

1
Q.14 x2 + y2  3x  3y + 4 = 0 Q.15 (1, 0) & (1/2,1/2); r =
2 2
Q.16 4x2 + 4y2 + 6x + 10y – 1 = 0 Q.17 40

Q.18 x2 + y2 – 12x – 12y + 64 = 0 Q.19 x2 + y2 ± a 2 x = 0


Q.20 (A) S; (B) R ; (C) Q ; (D) P

Get 10% Instant Discount On Unacademy Plus [Use Referral Code: MCSIRLIVE] 53
Maths IIT-JEE ‘Best Approach’ (MC SIR) Circle

EXERCISE–III
2
4 1
Q. 1 (a) D (b) B, C (c) c1 : (x – 4)2 + y2 = 9 c2 :  x   + y2 =
 3 9

common tangent between c & c1 : T1 = 0 ; T2 = 0 and x – 1 = 0 ;


common tangent between c & c2 : T1 = 0 ; T2 = 0 and x + 1 = 0

5  4
common tangent between c1 & c2 : T1 = 0 ; T2 = 0 and y = ± x  
39  5

where T1 : x – 3 y + 2 = 0 and T2 : x + 3y+2=0

Q.2 (a) C (b) A


Q.3 (a) 6 x  8 y + 25 = 0 & 6 x  8 y  25 = 0; (b) (–9/2 , 2)
(c) x2 + y2 + 4x – 12 = 0, T1: 3x  y  2 3  4  0 , T2: 3x  y  2 3  4  0 (D.C.T.)

T3: x  3 y  2  0 , T4: x  3 y  6  0 (T.C.T.)

Q.4 (a) A; (b) OA = 3(3 + 10 ) Q.5 (a) x2 + y2 + 14x – 6y + 6 = 0; (b) 2px + 2qy = r
Q.6 (a) C; (b) A Q.7 C
Q.8 2x2 + 2y2 – 10x – 5y + 1 = 0 Q.9 D Q.10 (a) B; (b) A
Q.11 (a) B; (b) C; (c) (i) D, (ii) A, (iii) D Q.12 B;
Q. 13 8 Q. 14 D Q. 15 2 Q. 16 A Q. 17 D
Q. 18 A Q. 19 D Q. 20 A, C Q. 21 A Q. 22 B, C
Q. 23 D Q. 24 D Q. 25 D Q. 26 B Q. 27 A
Q. 28 A, B, C Q. 29 A, C Q. 30 B, C, D Q. 31 2 Q. 32 A
Q. 33 D Q. 34 B, D Q.35 D Q.36 D Q.37 B
Q.38 C Q.39 C Q.40 C Q.41 B Q.42 B
Q.43 B Q.44 B Q.45 C Q.46 A Q.47 D
Q.48 C Q.49 B Q.50 B Q.51 A Q.52 B
Q.53 B Q.54 C Q.55 B Q.56 10.00 Q.57 C
Q.58 D Q.59 D Q.60 36 Q.61 9 Q.62 B
Q.63 2 Q.64 C Q.65 7 Q.66 D Q.67 2
Q.68 3 Q.69 56 Q.70 1225 Q.71 B Q.72 B
Q.73 A Q.74 9 Q.75 1 Q.76 C Q.77 A
Q.78 B Q.79 1 Q.80 B Q.81 C Q.82 C
Q.83 B Q.84 C Q.85 C Q.86 C Q.87 C
Q.88 A Q.89 A Q.90 C Q.91 16 Q.92 D
Q.93 61 Q.94 A Q.95 40 Q.96 165 Q.97 18
Q.98 B Q.99 D Q.100 B

Get 10% Instant Discount On Unacademy Plus [Use Referral Code: MCSIRLIVE] 54
Maths IIT-JEE ‘Best Approach’ (MC SIR) Circle
QUESTION BANK
Single Correct Type :

Q.1 Coordinates of the centre of the circle which bisects the circumferences of the circles
x2 + y2 = 1 ; x2 + y2 + 2x – 3 = 0 and x2 + y2 + 2y – 3 = 0 is
(A) (–1, 1) (B) (3, 3) (C) (2, 2) (D) (–2, –2)

*Q.2 The ends of a quadrant of a circle have the coordinates (1, 3) and (3, 1) then the centre of the such a
circle is
(A) (1, 1) (B) (2, 2) (C) (2, 6) (D) (4, 4)

* Q.3 Let C be a circle x2 + y2 = 1. The line l intersect C at the point (–1, 0) and the point P. Suppose that the
slope of the line l is a rational number m. Number of choices for m for which both the coordinates of P
are rational, is
(A) 3 (B) 4 (C) 5 (D) infinitely many

* Q.4 The line 2x – y + 1 = 0 is tangent to the circle at the point (2, 5) and the centre of the circles lies on
x – 2y = 4. The radius of the circle is
(A) 3 5 (B) 5 3 (C) 2 5 (D) 5 2

Q.5 One circle has a radius of 5 and its center at (0, 5). A second circle has a radius of 12 and its centre at
(12, 0). The length of a radius of a third circle which passes through the centre of the second circle and
both points of intersection of the first 2 circles, is equal to
(A) 13/2 (B) 15/2 (C) 17/2 (D) none

* Q.6 Consider 3 non collinear points A, B, C with coordinates (0, 6), (5, 5) and (–1, 1) respectively. Equation
of a line tangent to the circle circumscribing the triangle ABC and passing through the origin is
(A) 2x – 3y = 0 (B) 3x + 2y = 0
(C) 3x – 2y = 0 (D) 2x + 3y = 0

Q.7 A circle is inscribed in an equilateral triangle with side lengths 6 unit. Another circle is drawn inside the
triangle (but outside the first circle), tangent to the first circle and two of the sides of the triangle. The
radius of the smaller circle is
(A) 1 / 3 (B) 2/3 (C) 1/2 (D) 1

 3 3 
Q.8 To which of the following circles, the line y  x + 3 = 0 is normal at the point  3  , ?
 2 2
2 2 2 2
 3   3   3   3 
(A)  x  3   y   9 (B)  x   y   9
 2  2  2  2
(C) x2 + (y  3)2 = 9 (D) (x  3)2 + y2 = 9

Q.9 The circle with equation x2 + y2 = 1 intersects the line y = 7x + 5 at two distinct points A and B. Let C
be the point at which the positive x-axis intersects the circle. The angle ACB is
 4  3  3
(A) tan–1   (B) tan–1   (C) tan–1(1) (D) tan–1  
 3  4  2

Get 10% Instant Discount On Unacademy Plus [Use Referral Code: MCSIRLIVE] 55
Maths IIT-JEE ‘Best Approach’ (MC SIR) Circle
*Q.10 A square OABC is formed by line pairs xy = 0 and xy + 1 = x + y where ‘O’ is the origin. A circle with
centre C1 inside the square is drawn to touch the line pair xy = 0 and another circle with centre C2 and
radius twice that of C1, is drawn to touch the circle C1 and the other line pair. The radius of the circle with
centre C1 is
2 2 2 2 2 1
(A) (B) (C) (D)
3  2 1 3  2 1  3  2 1 3 2

*Q.11 If the circles


x2 + y2 + 2ax + 2by + c = 0
and x2 + y2 + 2bx + 2ay + c = 0
(a  b) 2
where c > 0, have exactly one point in common then the value of is
2c
(A) 1 (B) 2 (C) 2 (D) 1/2

*Q.12 The shortest distance from the line 3x + 4y = 25 to the circle x2 + y2 = 6x – 8y is equal to
(A) 7/5 (B) 9/5 (C) 11/5 (D) 32/5

*Q.13 Two circles with centres at A and B, touch at T. BD is the tangent at D and TC is a common tangent.
AT has length 3 and BT has length 2. The length CD is
(A) 4/3 (B) 3/2 (C) 5/3 (D) 7/4

B T A

C
D

Q.14 Triangle ABC is right angled at A. The circle with centre A and radius AB cuts BC and AC internally at
D and E respectively. If BD = 20 and DC = 16 then the length AC equals
(A) 6 21 (B) 6 26 (C) 30 (D) 32

Q.15 From the point A(0, 3) on the circle x2 + 4x + (y – 3)2 = 0 a chord AB is drawn & extended to a point
M such that AM = 2 AB. The equation of the locus of M is :
(A) x2 + 8x + y2 = 0 (B) x2 + 8x + (y – 3)2 = 0
(C) (x – 3)2 + 8x + y2 = 0 (D) x2 + 8x + 8y2 = 0

Q.16 If x = 3 is the chord of contact of the circle x2  y2 = 81, then the equation of the corresponding pair of
tangents, is
(A) x2  8y2 + 54x + 729 = 0 (B) x2  8y2  54x + 729 = 0
(C) x2  8y2  54x  729 = 0 (D) x2  8y2 = 729

Q.17 From (3, 4) chords are drawn to the circle x2 + y2 – 4x = 0. The locus of the mid points of the chords is
(A) x2 + y2 – 5x – 4y + 6 = 0 (B) x2 + y2 + 5x – 4y + 6 = 0
(C) x2 + y2 – 5x + 4y + 6 = 0 (D) x2 + y2 – 5x – 4y – 6 = 0

*Q.18 The centre of the smallest circle touching the circles x2 + y2 – 2y  3 = 0 and
x2 + y2  8x  18y + 93 = 0 is
(A) (3 , 2) (B) (4 , 4) (C) (2 , 7) (D) (2 , 5)

Get 10% Instant Discount On Unacademy Plus [Use Referral Code: MCSIRLIVE] 56
Maths IIT-JEE ‘Best Approach’ (MC SIR) Circle
Q.19 If the circle C1 : x2 + y2 = 16 intersects another circle C2 of radius 5 in such a manner that the
common chord is of maximum length and has a slope equal to 3/4, then the co-ordinates of the centre of
C2 are :
 9 12   9 12   12 9  12 9
(A)   ,   (B)   ,   (C)   ,  (D)   , 
 5 5  5 5  5 5  5 5

Q.20 In the diagram, DC is a diameter of the large circle centered at A, and AC is a diameter of the smaller
circle centered at B. If DE is tangent to the smaller circle at F and DC = 12 then the length of DE is
(A) 8 2 (B) 16 (C) 9 2 (D) 10 2

F E
D C
A B

Q.21 Let C be the circle of radius unity centred at the origin. If two positive numbers x1 and x2 are such that
the line passing through (x1, – 1) and (x2, 1) is tangent to C then
(A) x1x2 = 1 (B) x1x2 = – 1 (C) x1 + x2 = 1 (D) 4x1x2 = 1

Q.22 The locus of the centers of the circles which cut the circles x2 + y2 + 4x  6y + 9 = 0 and
x2 + y2  5x + 4y  2 = 0 orthogonally is :
(A) 9x + 10y  7 = 0 (B) x  y + 2 = 0
(C) 9x  10y + 11 = 0 (D) 9x + 10y + 7 = 0

Q.23 The locus of the center of the circles such that the point (2, 3) is the mid point of the chord 5x + 2y = 16
is
(A) 2x – 5y + 11 = 0 (B) 2x + 5y – 11 = 0 (C) 2x + 5y + 11 = 0 (D) none

Q.24 The locus of the mid points of the chords of the circle x² + y² + 4x  6y  12 = 0 which subtend an angle

of radians at its circumference is :
3
(A) (x  2)² + (y + 3)² = 6.25 (B) (x + 2)² + (y  3)² = 6.25
(C) (x + 2)² + (y  3)² = 18.75 (D) (x + 2)² + (y + 3)² = 18.75

Q.25 In a circle with centre ‘O’ PA and PB are two chords. PC is the chord that bisects the angle APB. The
tangent to the circle at C is drawn meeting PA and PB extended at Q and R respectively. If QC = 3,
QA = 2 and RC = 4, then length of RB equals
(A) 2 (B) 8/3 (C) 10/3 (D) 11/3

*Q.26 Suppose that two circles C1 and C2 in a plane have no points in common. Then
(A) there is no line tangent to both C1 and C2
(B) there are exactly four lines tangent to both C1 and C2
(C) there are no lines tangent to both C1 and C2 or there are exactly two lines tangent to both C1 and C2
(D) there are no lines tangent to both C1 and C2 or there are exactly four lines tangent to both C1 and C2

Q.27 If two chords of the circle x2 + y2 – ax – by = 0, drawn from the point (a, b) is divided by the x-axis in
the ratio 2 : 1 then :
(A) a2 > 3b2 (B) a2 < 3b2 (C) a2 > 4b2 (D) a2 < 4b2

Get 10% Instant Discount On Unacademy Plus [Use Referral Code: MCSIRLIVE] 57
Maths IIT-JEE ‘Best Approach’ (MC SIR) Circle
Q.28 Consider the points P(2, 1) ; Q(0, 0) ; R(4, –3) and the circle S : x 2 + y2 – 5x + 2y – 5 = 0
(A) exactly one point lies outside S (B) exactly two points lie outside S
(C) all the three points lie outside S (D) none of the point lies outside S

Q.29 The angle at which the circles (x – 1)2 + y2 = 10 and x2 + (y – 2)2 = 5 intersect is
   
(A) (B) (C) (D)
6 4 3 2

Q.30 P is a point (a, b) in the first quadrant. If the two circles which pass through P and touch both the
co-ordinate axes cut at right angles, then
(A) a2 – 6ab + b2 = 0 (B) a2 + 2ab – b2 = 0
(C) a2 – 4ab + b2 = 0 (D) a2 – 8ab + b2 = 0

Q.31 Three concentric circles of which the biggest is x2 + y2 = 1, have their radii in A.P. If the line y = x + 1 cuts
all the circles in real and distinct points. The interval in which the common difference of the A.P. will lie is

 1  1   2 2 
(A)  0,  (B)  0,  (C)  0,  (D) none
 4  2 2  4 

Q.32 A circle is inscribed into a rhombus ABCD with one angle 60°. The distance from the center of the circle
to the nearest vertex is equal to 1. If P is any point of the circle, then
|PA|2 + |PB|2 + |PC|2 + |PD|2 is equal to
(A) 12 (B) 11 (C) 9 (D) none

*Q.33 The value of 'c' for which the set, {(x, y)x2 + y2 + 2x  1}  {(x, y)x  y + c  0} contains only
one point in common is :
(A) (, 1]  [3, )(B) {1, 3} (C) {3} (D) { 1 }

Q.34 A tangent at a point on the circle x2 + y2 = a2 intersects a concentric circle C at two points P and Q. The
tangents to the circle C at P and Q meet at a point on the circle x2 + y2 = b2 then the equation of circle
‘C’ is
(A) x2 + y2 = ab (B) x2 + y2 = (a – b)2
2 2
(C) x + y = (a + b) 2 (D) x2 + y2 = a2 + b2

Q.35 Tangents are drawn to the circle x2 + y2 = 1 at the points where it is met by the circles,
x2 + y2 – ( + 6)x + (8 – 2)y – 3 = 0 .  being the variable. The locus of the point of intersection of
these tangents is :
(A) 2x – y + 10 = 0 (B) x + 2y – 10 = 0 (C) x – 2y + 10 = 0 (D) 2x + y – 10 = 0

 1  1  1  1
Q.36 If  a ,  ,  b ,  ,  c ,  and  d ,  are four distinct points on a circle of radius 4 units then,
 a  b  c  d
abcd is equal to
(A) 4 (B) 1/4 (C) 1 (D) 16

Q.37 ABCD is a square of unit area. A circle is tangent to two sides of ABCD and passes through exactly one
of its vertices. The radius of the circle is
1 1
(A) 2  2 (B) 2 1 (C) (D)
2 2

Get 10% Instant Discount On Unacademy Plus [Use Referral Code: MCSIRLIVE] 58
Maths IIT-JEE ‘Best Approach’ (MC SIR) Circle
Q.38 A pair of tangents are drawn to a unit circle with centre at the origin and these tangents intersect at A
enclosing an angle of 60°. The area enclosed by these tangents and the arc of the circle is
2    3  
(A)  (B) 3  (C)  (D) 3  1  
3 6 3 3 6  6

Q.39 A straight line with slope 2 and y-intersect 5 touches the circle, x2 + y2 + 16x + 12y + c = 0 at a point Q.
Then the coordinates of Q are
(A) (–6, 11) (B) (–9, –13) (C) (–10, –15) (D) (–6, –7)

Q.40 A foot of the normal from the point (4, 3) to a circle is (2, 1) and a diameter of the circle has the equation
2x – y – 2 = 0. Then the equation of the circle is
(A) x2 + y2 – 4y + 2 = 0 (B) x2 + y2 – 4y + 1 = 0
2 2
(C) x + y – 2x – 1 = 0 (D) x2 + y2 – 2x + 1 = 0

Q.41 AB is a diameter of a circle. CD is a chord parallel to AB and 2 CD = AB. The tangent at B meets the
line AC produced at E then AE is equal to :
(A) AB (B) 2 AB (C) 2 2 AB (D) 2 AB

Q.42 A circle of constant radius ' a ' passes through origin ' O ' and cuts the axes of coordinates in points P
and Q, then the equation of the locus of the foot of perpendicular from O to PQ is :

(A) (x2 + y2)  12  12  = 4 a2 (B) (x2 + y2)2  12  12  = a2


x y  x y 

(C) (x2 + y2)2  12  12  = 4 a2 (D) (x2 + y2)  12  12  = a2


x y  x y 

Q.43 If a circle of constant radius 3k passes through the origin 'O' and meets co-ordinate axes at A and B
then the locus of the centroid of the triangle OAB is
(A) x2 + y2 = (2k)2 (B) x2 + y2 = (3k)2 (C) x2 + y2 = (4k)2 (D) x2 + y2 = (6k)2

Q.44 Tangents are drawn from (4, 4) to the circle x2 + y2 – 2x – 2y – 7 = 0 to meet the circle at A and B. The
length of the chord AB is
(A) 2 3 (B) 3 2 (C) 2 6 (D) 6 2

Q.45 Points P and Q are 3 units apart. A circle centre at P with a radius of 3 units intersects a circle centred at
Q with a radius of 3 units at point A and B. The area of the quadrilateral APBQ is
99 99 99
(A) 99 (B) (C) (D)
2 2 16

Q.46 Tangents are drawn from any point on the circle x2 + y2 = R2 to the circle x2 + y2 = r2. If the line joining
the points of intersection of these tangents with the first circle also touch the second, then R equals
2r 4r
(A) 2r (B) 2r (C) (D)
2 3 3 5

Q.47 The equation of the circle symmetric to the circle x2 + y2 – 2x – 4y + 4 = 0 about the line x – y = 3 is
(A) x2 + y2 – 10x + 4y + 28 = 0 (B) x2 + y2 + 6x + 8 = 0
(C) x2 + y2 – 14x – 2y + 49 = 0 (D) x2 + y2 + 8x + 2y + 16 = 0

Get 10% Instant Discount On Unacademy Plus [Use Referral Code: MCSIRLIVE] 59
Maths IIT-JEE ‘Best Approach’ (MC SIR) Circle
Q.48 The locus of the centre of a circle which touches externally the circle, x2 + y2 – 6x – 6y + 14 = 0 & also
touches the y-axis is given by the equation :
(A) x2 – 6x – 10y + 14 = 0 (B) x2 – 10x – 6y + 14 = 0
(C) y2 – 6x – 10y + 14 = 0 (D) y2 – 10x – 6y + 14 = 0

Q.49 The equation of the locus of the mid points of the chords of the circle 4x2 + 4y2 – 12x + 4y + 1 = 0 that
2
subtend an angle of at its centre is
3
(A) 16(x2 + y2) – 48x + 16y + 31 = 0 (B) 16(x2 + y2) – 48x – 16y + 31 = 0
(C) 16(x2 + y2) + 48x + 16y + 31 = 0 (D) 16(x2 + y2) + 48x – 16y + 31 = 0

Q.50 Number of value(s) of A for which the system of equations x2 = y2 and (x – A)2 + y2 = 1 has exactly 3
solutions, is
(A) 1 (B) 2 (C) 3 (D) 4

Q.51 A variable circle C has the equation


x2 + y2 – 2(t2 – 3t + 1)x – 2(t2 + 2t)y + t = 0, where t is a parameter.
If the power of point P(a,b) w.r.t. the circle C is constant then the ordered pair (a, b) is
1 1  1 1 1 1  1 1
(A)  ,   (B)   ,  (C)  ,  (D)   ,  
 10 10   10 10   10 10   10 10 

Q.52 Consider the circles, x2 + y2 = 25 and x2 + y2 = 9. From the point A(0, 5) two segments are drawn
touching the inner circle at the points B and C while intersecting the outer circle at the points D and E. If
‘O’ is the centre of both the circles then the length of the segment OF that is perpendicular to DE, is
(A) 7/5 (B) 7/2 (C) 5/2 (D) 3

Paragraph for Question Nos. 53 to 55

Let C be a circle of radius r with centre at O. Let P be a point outside C and D be a point on C. A line
through P intersects C at Q and R, S is the midpoint of QR.
Q.53 For different choices of line through P, the curve on which S lies, is
(A) a straight line (B) an arc of circle with P as centre
(C) an arc of circle with PS as diameter (D) an arc of circle with OP as diameter

Q.54 Let P is situated at a distance 'd' from centre O, then which of the following does not equal the product
(PQ) (PR)?
(A) d2 – r2 (B) PT2, where T is a point on C and PT is tangent to C
(C) (PS)2 – (QS)(RS) (D) (PS)2

Q.55 Let XYZ be an equilateral triangle inscribed in C. If , ,  denote the distances of D from vertices X,
Y, Z respectively, the value of product ( +  – ) ( +  – ) ( +  – ), is

(A) 0 (B)
8

 3  3   3  3
(C) (D) None of these
6

Get 10% Instant Discount On Unacademy Plus [Use Referral Code: MCSIRLIVE] 60
Maths IIT-JEE ‘Best Approach’ (MC SIR) Circle
Paragraph for Question Nos. 56 to 58
Consider the circle S : x2 + y2 – 4x – 1 = 0 and the line L : y = 3x – 1. If the line L cuts the circle at A and
B then

Q.56 Length of the chord AB equal


(A) 2 5 (B) 5 (C) 5 2 (D) 10

Q.57 The angle subtended by the chord AB in the minor arc of S is


3 5 2 
(A) (B) (C) (D)
4 6 3 4

Q.58 Acute angle between the line L and the circle S is


   
(A) (B) (C) (D)
2 3 4 6
Assertion & Reason
Q.59 Let A(x1, y1), B(x2, y2) and C(x3, y3) are the vertices of a triangle ABC.
Statement-1 : If angle C is obtuse then the quantity (x3 – x1)(x3 – x2) + (y3 – y1)(y3 – y2) is negative.
because
Statement-2 : Diameter of a circle subtends obtuse angle at any point lying inside the semicircle.
(A) Statement-1 is true, Statement-2 is true and Statement-2 is correct explanation for Statement-1.
(B) Statement-1 is true, Statement-2 is true and Statement-2 is NOT the correct explanation for Statement-1.
(C) Statement-1 is true, Statement-2 is false
(D) Statement-1 is false, Statement-2 is true
Q.60 Statement-1 : Angle between the tangents drawn from the point P(13, 6) to the circle
S : x2 + y2 – 6x + 8y – 75 = 0 is 90°
because
Statement-2 : Point P lies on the director circle of S.
(A) Statement-1 is true, Statement-2 is true and Statement-2 is correct explanation for Statement-1.
(B) Statement-1 is true, Statement-2 is true and Statement-2 is NOT the correct explanation for Statement-1.
(C) Statement-1 is true, Statement-2 is false
(D) Statement-1 is false, Statement-2 is true
Q.61 Consider the circle C : x2 + y2 – 2x – 2y – 23 = 0 and a point P(3, 4).
Statement-1 : No normal can be drawn to the circle C, passing through (3, 4).
because
Statement-2 : Point P lies inside the given circle, C.
(A) Statement-1 is true, Statement-2 is true and Statement-2 is correct explanation for Statement-1.
(B) Statement-1 is true, Statement-2 is true and Statement-2 is NOT the correct explanation for Statement-1.
(C) Statement-1 is true, Statement-2 is false
(D) Statement-1 is false, Statement-2 is true
Q.62 Consider the lines
L : (k + 7)x – (k – 1)y – 4(k – 5) = 0 where k is a parameter
and the circle
C : x2 + y2 + 4x + 12y – 60 = 0
Statement-1 : Every member of L intersects the circle ‘C’ at an angle 90°
because
Statement-2 : Every member of L is tangent to the circle C.
(A) Statement-1 is true, Statement-2 is true and Statement-2 is correct explanation for Statement-1.
(B) Statement-1 is true, Statement-2 is true and Statement-2 is NOT the correct explanation for Statement-1.
(C) Statement-1 is true, Statement-2 is false
(D) Statement-1 is false, Statement-2 is true

Get 10% Instant Discount On Unacademy Plus [Use Referral Code: MCSIRLIVE] 61
Maths IIT-JEE ‘Best Approach’ (MC SIR) Circle
One or more than one correct :
Q.63 The equation of a circle with centre (4, 3) and touching the circle x2 + y2 = 1 is :
(A) x2 + y2 – 8x – 6y – 9 = 0 (B) x2 + y2 – 8x – 6y + 11 = 0
(C) x2 + y2 – 8x – 6y – 11 = 0 (D) x2 + y2 – 8x – 6y + 9 = 0

Q.64 A circle passes through the points (–1, 1), (0, 6) and (5, 5). The point(s) on this circle, the tangent(s) at
which is/are parallel to the straight line joining the origin to its centre is/are :
(A) (1, –5) (B) (5, 1) (C) (–5, –1) (D) (–1, 5)

*Q.65 Point M moved along the circle (x – 4)2 + (y – 8)2 = 20. Then it broke away from it and moving along
a tangent to the circle, cuts the x-axis at the point (–2, 0). The co-ordinates of the point on the circle at
which the moving point broke away can be :
 3 46   2 44 
(A)   ,  (B)   ,  (C) (6, 4) (D) (3, 5)
 5 5   5 5 

Q.66 The centre(s) of the circle(s) passing through the points (0, 0), (1, 0) and touching the circle x2 + y2 = 9
is/are
3 1 1 3  1 1/2  1 1/2 
(A)  ,  (B)  ,  (C)  , 2  (D)  , 2 
2 2 2 2 2  2 

Q.67 Which of the following lines have the intercepts of equal lengths on the circle, x2 + y2 – 2x + 4y = 0 ?
(A) 3x – y = 0 (B) x + 3y = 0 (C) x + 3y + 10 = 0 (D) 3x – y – 10 = 0

*Q.68 Consider the circles C1 : x2 + y2 = 16 and C2 : x2 + y2 – 12x + 32 = 0. Which of the following statements
is/are correct ?
(A) Number of common tangent to these circles is 3
(B) The point P with coordinates (4, 1) lies outside the circle C1 and inside the C2
(C) Their direct common tangent intersect at (12, 0)
(D) Slope of their radical axis is not defined

Q.69 Consider the circles


C1 : x2 + y2 – 4x + 6y + 8 = 0
C2 : x2 + y2 – 10x – 6y + 14 = 0
Which of the following statement(s) hold good in respect of C1 and C2 ?
(A) C1 and C2 are orthogonal
(B) C1 and C2 touch each other
(C) Radical axis between C1 and C2 is also one of their common tangent
(D) Middle point of the line joining the centres of C1 and C2 lies on their radical axis

*Q.70 Three distinct lines are drawn in a plane. Suppose there exist exactly n circles in the plane tangent to all
the three lines, then the possible values of n is/are
(A) 0 (B) 1 (C) 2 (D) 4

Get 10% Instant Discount On Unacademy Plus [Use Referral Code: MCSIRLIVE] 62
Maths IIT-JEE ‘Best Approach’ (MC SIR) Circle
ANSWER KEY
Single Correct :

Q.1 D Q.2 A Q.3 D Q.4 A Q.5 A Q.6 D Q.7 A


Q.8 D Q.9 C Q.10 C Q.11 A Q.12 A Q.13 B Q.14 B
Q.15 B Q.16 B Q.17 A Q.18 D Q.19 B Q.20 A Q.21 A
Q.22 C Q.23 A Q.24 B Q.25 B Q.26 D Q.27 A Q.28 D
Q.29 B Q.30 C Q.31 C Q.32 B Q.33 D Q.34 A Q.35 A
Q.36 C Q.37 A Q.38 B Q.39 D Q.40 C Q.41 D Q.42 C
Q.43 A Q.44 B Q.45 B Q.46 B Q.47 A Q.48 D Q.49 A
Q.50 B Q.51 B Q.52 A Q.53 D Q.54 D Q.55 A Q.56 D
Q.57 A Q.58 C Q.59 A Q.60 A Q.61 D Q.62 C

One or more than one correct

Q.63 CD Q.64 BD Q.65 BC Q.66 CD Q.67 ABCD Q.68 ACD


Q.69 BC Q.70 ACD

Get 10% Instant Discount On Unacademy Plus [Use Referral Code: MCSIRLIVE] 63

You might also like